Neuro Flashcards
Which of the following features describes an intra-axial mass? [B3 Q14]
A. Cortex displaced towards bone
B. Mass contiguous with dura
C. Buckling of grey and white matter
D. Widened subarachnoid cistern
E. Dural feeding arteries
A
B-E are extra-axial features
A mass is seen peripherally in the middle cranial fossa on MR of the brain. Which of the following imaging features favours an intra-axial rather than an extra-axial location? [B4 Q17]
a. buckling of the grey–white matter interface
b. expansion of the cortex of the brain
c. expansion of the subarachnoid space
d. medial displacement of pial blood vessels by the mass
e. the mass has a dural base
b. expansion of the cortex of the brain
Once the presence of a mass has been established, the radiologist must determine whether the mass is intra-axial (arising within the brain parenchyma) or extra-axial (arising outside the brain substance) to formulate an appropriate differential diagnosis. An extra-axial mass characteristically causes buckling of the grey–white matter interface, expansion of the subarachnoid space at its borders, and medial displacement of the vessels in the subarachnoid space. A dural base is also a feature of an extra-axial mass. Intra-axial masses characteristically cause expansion of the cortex of the brain but no expansion of the subarachnoid space, and pial vessels may be seen peripheral to the mass.
Which of the following is the most common radiation-induced CNS tumour? [B4 Q60] {will repeat again in respective tumour section}
a. ependymoma
b. oligodendroglioma
c. lymphoma
d. glioblastoma multiforme
e. meningioma
e. meningioma
Meningioma is the most common radiation-induced CNS tumour and has been particularly associated with low-dose radiation treatment for tinea capitis. For the diagnosis of radiation-induced meningioma to be made, the meningioma must arise in the radiation field, appear after a latency period of years and should not have been the primary tumour irradiated. Radiation-induced meningiomas are more frequently multiple and have higher recurrence rates than non-radiation-induced tumours.
A 40-year-old man is being investigated by the neurologists for new onset epilepsy. An electroencephalogram (EEG) indicates an epileptogenic focus in the left temporal lobe. An MRI is carried out. The hippocampal structures are unremarkable. A 2-cm lesion is noted in the subcortical region of the left temporal lobe. This lesion demonstrates mild enhancement. Areas of low signal on T1WI and T2WI are felt to represent foci of calcification. There is also an area of high signal on T1WI and T2WI seen inferiorly within the lesion, which probably represents an area of haemorrhage. There is a rim of surrounding oedema noted on T2WI and fluid-attenuated inversion recovery (FLAIR). What is the most likely diagnosis? [B1 Q5]
A. Haemangioblastoma.
B. Desmoplastic infantile ganglioglioma (DIG).
C. Dysembryoplastic neuroepithelial tumour (DNET).
D. Pleomorphic xanthoastrocytoma (PXA).
E. Oligodendroglioma.
E. Oligodendroglioma.
Prior to evaluating the imaging characteristics, the patient’s demographics should be considered in this case, as in all cases of intracranial masses, as it will help limit the differential diagnosis significantly. The patient is an adult, thus making DIG and DNET unlikely. Secondly, consider the temporal lobe location. Haemangioblastomas are usually infratentorial in location, especially in the absence of a history of von Hippel–Lindau syndrome. All the other lesions are classical temporal lobe tumours. Finally, considering the imaging characteristics, both PXA and oligodendrogliomas may be entirely solid, but cysts are commonly seen in PXAs. Calcification is seen in 60–80% of oligodendrogliomas but is rarely seen in PXAs. Similarly, haemorrhage, while not typical of oligodendrogliomas, is rare in PXAs.
An 8-year-old female patient presents to your paediatric neurology service with a history of increasing ataxia, repeated headaches, and vomiting, increasing in severity over the last 5 months. Clinical examination reveals marked cerebellar signs of past pointing and dysdiadochokinesis. An MRI is requested, which shows a solid mass in the posterior fossa measuring 2 cm in size. This mass arises in the left cerebellar hemisphere and displaces the fourth ventricle. It is of low intensity on T1WI and high signal on T2WI. There is only a small rim of surrounding oedema. The lesion demonstrates relatively homogeneous moderate enhancement. There is no evidence of subarachnoid seeding. What is the most likely diagnosis? [B1 Q14]
A. Pilocytic astrocytoma.
B. Ependymoma.
C. Medulloblastoma.
D. Metastasis.
E. Lhermitte–Duclos syndrome
A. Pilocytic astrocytoma.
This question deals with the classical neurological differential diagnosis of a posterior fossa mass in a child. While there are many causes, pilocytic astrocytoma and medulloblastoma account for over 60% of all childhood posterior fossa masses. Pilocytic astrocytomas have a classical appearance of being cystic lesions with an avidly enhancing mural nodule. However, 30% of pilocytic astrocytomas are solid tumours. In differentiating them from medulloblastomas, pilocytic astrocytomas often arise more peripherally and displace the fourth ventricle, whereas medulloblastomas usually arise centrally from the vermis. Subarachnoid seeding is seen in up to 50% of cases of medulloblastoma. Ependymomas are also included in the differential. As these arise from the ependyma lining the ventricle, they tend to be centred on the fourth ventricle in children. Metastases are the most common cause of a posterior fossa mass in adults but are less common in children.
A patient is having an MRI scan carried out to investigate a possible right frontal astrocytoma, incidentally detected on CT following a head injury. The MRI features are typical of an astrocytoma, with no evidence of necrosis or callosal involvement to indicate glioblastoma multiforme (GBM). MRS has been carried out to help assess the grade of this tumour. What MRS features would indicate a high-grade lesion? [B1 Q17]
A. Elevated choline, reduced N-acetyl aspartate (NAA), choline/creatine (Cho/Cr) ratio of 1.
B. Elevated choline, reduced NAA, Cho/Cr ratio of 2.
C. Normal choline, elevated NAA.
D. Reduced choline, reduced NAA. Cho/Cr ratio of 1.2.
E. All normal, these are unaffected by tumour grade.
B. Elevated choline, reduced NAA, Cho/Cr ratio of 2.
NAA is thought to be a marker of neuronal integrity, choline indicates cell turnover, and creatine indicates cell metabolism. Lactate is not detectable in normal brain spectra but is elevated in inflammation, infarction, and some neoplasms. Most brain conditions, whether neoplastic, vascular, or demyelinating, are associated with a reduction in NAA. A notable exception is Canavan’s disease, which causes a rise in NAA. Choline is elevated in many disorders but is markedly increased in high-grade neoplasms. It has been reported that the ratio of choline to creatine can be used to help grade tumours, with a ratio over 1.5 indicating high grade in most cases. A reduced choline and NAA in an area of tumour can indicate necrosis.
A 45-year-old woman presents with a several-month history of neck pain and gradually progressive weakness and paraesthesia in the upper limbs. An MRI scan of the cervical spine is performed, and this shows a well-defined central intramedullary mass in the mid-cervical spinal cord. The mass is generally slightly hyperintense on T2WI, but also has a few low signal peripheral areas. It enhances homogeneously with gadolinium. What is the most likely diagnosis? [B1 Q27] {will repeat again in Spinal Cord Tumour Section}
A. Astrocytoma.
B. Metastasis.
C. Haemangioblastoma.
D. Ganglioglioma.
E. Ependymoma.
E. Ependymoma.
Ependymoma is the most common intramedullary neoplasm in adults. It tends to be centrally located within the cord, unlike astrocytoma, which can be eccentric. Astrocytoma can have a longer segment of cord involvement than ependymoma and may have a more infiltrative margin. The peripheral low signal areas seen on T2WI in ependymoma are related to haemosiderin deposition from prior haemorrhage. Haemangioblastoma is more often seen in the dorsal cord than the cervical cord and is typically a small well-defined lesion. It may have an associated cord cyst or syrinx. Flow voids may be seen within the lesion, from dilated vascular channels. Ganglioglioma is a very rare, slow-growing tumour of low malignant potential. The imaging appearance is non-specific, but there are some findings that may suggest the diagnosis. Compared with other spinal cord tumors, gangliogliomas are more likely to involve long segments of the cord (greater than four levels, up to the whole cord), to be associated with bone erosion or scalloping, to have tumoral cysts, and to have areas of mixed high signal on precontrast T1WI. Intramedullary metastasis represents less than 5% of intramedullary lesions. They usually occur in the setting of advanced malignant disease, typically from a lung or breast primary. The spinal cord oedema can seem out of keeping with the small size of the metastatic lesion.
When on call you are asked to perform a CT head scan for a 17-year-old male who presents with seizures. He is unable to provide a history. A look on the computer system shows that he has had previous regular abdominal ultrasounds and an echocardiogram as a child. Brain CT shows a hypodense, well-demarcated, rounded mass in the region of the foramen of Monro. It is partially calcified, and it demonstrates uniform enhancement. What is the most likely diagnosis? [B2 Q4] {will repeat again in periventricular tumour section}
a. Colloid cyst
b. Giant cell astrocytoma
c. Metastasis
d. Lymphoma
e. Haemangioblastoma
Giant cell astrocytoma
The main differential for a mass at the foramen of Monro is between a colloid cyst and a subependymal giant cell astrocytoma. The latter is associated with tuberous sclerosis (TS). Renal involvement is also relatively common in TS and patients regularly have surveillance renal ultrasounds. The echocardiogram was performed to assess for cardiomyopathy/rhabdomyoma. Other central nervous system findings in patients with TS include: 1. Subependymal hamartomas – these are nodular and irregular and can be located anywhere along the ventricular walls but predominantly occur around the foramen of Monro or along the lateral ventricles. In infants, with unmyelinated white matter the lesions are usually hyperintense on T1 and hypointense on T2. The reverse is seen in adults. 2. Subcortical and cortical hamartomas (tubers) – these appear as broad cortical gyri with abnormalities in the adjacent white matter. They frequently calcify but enhancement is extremely rare. 3. Heterotopic grey matter islands in white matter. These may calcify and show contrast enhancement.
A 40-year-old man undergoes investigation for seizures. Head CT with and without contrast shows a large, round, sharply marginated, hypodense mass involving the cortex and subcortical white matter of the left frontal lobe. The mass contains large nodular clumps of calcification. There is surrounding oedema and ill-defined enhancement. MRI demonstrates a heterogeneous mass which is predominantly isointense to grey matter on T1 and hyperintense on T2. There is moderate enhancement. What is the most likely diagnosis? [B2 Q20]
a. Astrocytoma
b. Ganglioglioma
c. Ependymoma
d. Glioblastoma
e. Oligodendroglioma
Oligodendroglioma
This is an uncommon glioma which usually presents as a large mass at the time of diagnosis. Mean age is 30–50 years and they are more common in men than women. The majority are in the frontal lobe (60%), although they can occur anywhere within the central nervous system, including the cerebellum, brainstem, spinal cord, ventricles, and optic nerve. Large nodular clumps of calcifications are present in up to approximately 90% of tumours. Cystic degeneration and haemorrhage are uncommon. Prognosis depends on the grade of the tumour. High-grade tumours show 20% ten-year survival whereas low-grade tumours show 46% ten-year survival. Although astrocytomas can calcify, the calcifications are rarely large and nodular. Glioblastomas rarely calcify. Gangliogliomas are more common in the temporal lobes and deep cerebral tissues and most of them (80%) occur below the age of 30 years. Ependymomas often demonstrate fluid levels due to internal haemorrhage.
A 36-year-old female presents following a tonic-clonic seizure. Over the preceding months she had suffered with progressive, severe headaches. Contrast-enhanced CT brain shows lateral displacement of the internal capsules by enlarged thalami but no abnormal enhancement. T2-weighted MRI demonstrates a diffuse, contiguous area of hyperintensity involving the thalami, caudate and lentiform nuclei, the splenium of the corpus callosum and the periventricular white matter. There is only minimal mass effect. T1-weighted gadolinium imaging shows no enhancement. What is the most likely diagnosis? [B2 Q50]
a. Multiple sclerosis
b. Gliomatosis cerebri
c. Viral encephalitis
d. Adrenoleukodystrophy
e. Vasculitis
Gliomatosis cerebri (GC)
GC is a diffusely infiltrative glioma that may be present with or without a dominant mass. It must, however, involve two or more lobes and usually involves contiguous areas. It affects all age groups and can be of varying histological grade. Presentation may be enigmatic as the normal cerebral architecture is usually preserved. Alternatively, patients present with seizures, headache and personality disorders. Prognosis is poor. MRI findings include diffuse T2 (and proton density) hyperintensity throughout the white matter that usually extends to involve the deep grey nuclei with enlargement of cerebral structures. It is often bilateral and symmetric with minor mass effect and absence of necrosis. The differential diagnosis of symmetric white matter lesions includes microvascular change, encephalitis, demyelinating disease, vasculitis, and leukoencephalopathy. GC is the most likely diagnosis in this scenario as there is involvement of the corpus callosum and the pattern is infiltrative with enlargement of the thalami (cerebral structures).
Which brain tumour has the greatest incidence across all age groups? [B3 Q9]
A. Glioma
B. Meningioma
C. Metastases
D. Pituitary adenoma
E. Haemangioblastoma
Glioma
Gliomas consist of astrocytoma, oligodendrogliomas, paragangliomas, gangliogliomas and medulloblastomas.
Which is the most common location of oligodendroglioma? [B3 Q23]
A. Frontal lobe
B. Temporal lobe
C. Parietal lobe
D. Occipital lobe
E. Cerebellum
Frontal
Most commonly involve cortical and subcortical white matter, occasionally through corpus callosum as butterfly glioma.
Which of the following conditions will typically demonstrate unrestricted diffusion on MR DWI and ADC map? [B4 Q6]
a. epidermoid cyst
b. acute infarction
c. cerebral abscess
d. glioblastoma multiforme
e. viral encephalitis
Glioblastoma Multiforme
Diffusion-weighted MRI provides image contrast which is different from that provided by conventional MR techniques. The sequence enables the measurement of net macroscopic water movement, which is anisotropic (varies in different directions) particularly in white matter. Restricted diffusion is seen as high signal on DWI (which is a T2W image with signal degraded by diffusion) and low signal on the ADC map. Restricted diffusion occurs in tissue that does not allow free movement of water molecules, such as areas of infection due to the high viscosity and cellularity of pus. Similarly, epidermoid cysts are very cellular and so also show restricted diffusion, a feature that helps distinguish them from arachnoid cysts, which are fluid structures. In stroke, restriction in water diffusion occurs within minutes after the onset of ischaemia. The basis of this change is not completely clear but is thought to be related to the cytotoxic oedema seen in ischaemic cells due to the impairment of the Na+/K+ ATPase pumps (which are very energy dependent), leading to loss of ionic gradients and a net translocation of water from the extracellular to the intracellular compartment, where water mobility is relatively more restricted.
A 46-year-old female presents with back pain and increasing weakness of the lower limbs. An MR scan shows a lesion in the cord at the level of T11. Which of the following features would suggest an ependymoma rather than demyelination? [B4 Q16]
a. multiple lesions
b. expansion of the cord
c. high signal on T2W images
d. enhancement with gadolinium
e. peripheral low signal on all sequences
Expansion of the cord
Ependymomas are the commonest tumour of the spinal cord in adults, accounting for 40–60% of cord tumours. They present with a long history of pain, and sensory or motor disturbance. Less commonly, bladder and bowel dysfunction may occur. Expansion of the cord is more often seen with ependymomas than with demyelination. Both lesions may enhance and have high signal on T2W images, but multiplicity is more often seen with demyelination. Peripheral low signal, usually indicating haemosiderin, is not a feature of either of these lesions.
A 5-year-old boy undergoes CT of the brain for investigation of headaches, vomiting and ataxia. This demonstrates a well-defined, multilobulated, iso-dense mass within the fourth ventricle containing areas of punctate calcification. The mass is seen to extend out of the foramina of Luschka into the cerebellopontine angles. There is associated hydrocephalus. What is the most likely diagnosis? [B4 Q31]
a. metastasis
b. haemangioblastoma
c. juvenile pilocytic astrocytoma
d. medulloblastoma
e. ependymoma
Ependymoma
Ependymomas most commonly arise in the floor of the fourth ventricle and are usually isodense. They have a greater incidence of calcification than other posterior fossa paediatric tumours; it is typically punctate and seen in 40–50% of cases. A characteristic feature of ependymomas is their propensity to extend through and widen the foramina of Luschka and Magendie. Juvenile pilocytic astrocytomas are the commonest paediatric infratentorial neoplasms and typically occur in the cerebellar hemispheres. They appear cystic with an enhancing mural nodule. Medulloblastomas tend to be homogeneous hyperdense lesions located in the vermis, and the presence of calcification is uncommon. Metastases are the commonest infratentorial tumour to occur in adults but are uncommon in children. Haemangioblastomas usually occur in young adults and are classically cystic masses with a solid mural nodule.
A 65-year-old, previously well man with a short history of headaches and behavioural change undergoes CT of the brain. This demonstrates an irregular, ill-defined mass in the left frontal lobe extending across the corpus callosum to involve the right frontal lobe. The mass is of low attenuation and contains cystic areas, demonstrates ring enhancement following intravenous contrast, and has considerable surrounding oedema. What is the most likely diagnosis? [B4 Q39]
a. progressive multifocal leukoencephalopathy
b. glioblastoma multiforme
c. lymphoma
d. abscess
e. metastasis
Glioblastoma Multiforme
Glioblastoma multiforme is the most malignant form of astrocytoma. It occurs in older patients, and most commonly affects the deep white matter of the frontal lobes. Classic appearances are of an irregular, ill-defined hypodense mass with necrosis, haemorrhage and extensive surrounding white matter oedema. Ninety per cent of cases show enhancement, which may be diffuse, heterogeneous or ring-like. Tumour spread is directly along white matter tracts, and commonly occurs across the corpus callosum to involve both frontal lobes (butterfly glioma). Lymphomas also have a propensity to involve the corpus callosum but usually are slightly hyperdense due to a high nuclearto-cytoplasmic ratio. Metastases may also involve the corpus callosum.
A 28-year-old woman presents with a history of headaches and refractory temporal lobe epilepsy. CT of the brain demonstrates a mixed solid–cystic, intraparenchymal mass located peripherally in the right temporal lobe, which contains calcification and demonstrates faint enhancement following intravenous contrast. There is minimal surrounding oedema. What is the most likely diagnosis? [B4 Q47]
a. arachnoid cyst
b. ganglioglioma
c. epidermoid
d. meningioma
e. dysembryoplastic neuroepithelial tumour
Ganglioglioma
Gangliogliomas are low-grade tumours with a good prognosis, generally occurring in patients under the age of 30. Typical presentation is with focal seizures, and ganglioglioma is the most common tumour seen in patients with chronic temporal lobe epilepsy. They are usually well circumscribed, hypo- or iso-dense lesions in the temporal lobes. Calcification (30%) and cyst formation (50%) are common features. There is usually minimal mass effect and surrounding oedema. Meningiomas commonly calcify and have minimal surrounding oedema, but are extra-axial, and usually demonstrate intense uniform enhancement following intravenous contrast. Dysembryoplastic neuroepithelial tumours are commonly associated with partial complex seizures, but usually occur before the age of 20, and characteristically appear as a soap-bubble, multicystic lesion, which may remodel the calvarium. Epidermoid and arachnoid cysts are of CSF density, do not enhance with contrast and are extra-axial lesions.
A 44-year-old man presents with a long history of headaches and more recent onset of seizures. CT of the brain demonstrates an oval, well-defined, heterogeneous, hypodense mass containing large nodular clumps of calcification located peripherally in the right frontal lobe. The mass extends to the cortical margin, and there is erosion of the inner table of the skull. There is minimal surrounding vasogenic oedema. What is the most likely diagnosis? [B4 Q50]
a. meningioma
b. oligodendroglioma
c. astrocytoma
d. glioblastoma multiforme
e. ganglioglioma
Oligodendroglioma
Oligodendrogliomas are slow-growing tumours, usually presenting in adults aged 30–50 years. They occur most commonly in the frontal lobe, often extending to the cortex and eroding the inner table of the skull. Calcification is seen in 70% of cases, typically appearing as large nodular clumps, with a relative absence of surrounding oedema. Astrocytomas also appear as hypodense calcified lesions but lack calvarial erosion. Glioblastoma multiforme typically has considerable surrounding oedema and rarely calcifies. Gangliogliomas show calcification in a third of cases but tend to occur in younger patients and have a predilection for the temporal lobes.
A 32-year-old man with a 3-month history of headaches presents to the Accident & Emergency Department with tonic-clonic seizures. MRI shows a 5 cm intra-axial lesion in the left frontal lobe. The lesion appears hypointense on T1 and hyperintense on T2 to brain parenchyma. No significant surrounding oedema is seen and there is no enhancement with gadolinium. The most likely diagnosis is? [B5 Q24]
(a) Oligodendroglioma
(b) Astrocytoma
(c) Arachnoid cyst
(d) Metastases
(e) Lymphoma
Astrocytoma
These MRI appearances are typical of a grade II astrocytoma. Grade III are more infiltrative and show more surrounding oedema. Oligodendrogliomas show calcifications. Arachnoid cysts show CSF density on all sequences. Metastatic lesions and lymphoma enhance with gadolinium.
A 42-year-old man presents with increasing headache and blurred vision. CT of the head shows a large lesion in the periphery of the left parietal lobe with extensive calcification. The lesion shows heterogeneous contrast enhancement. There is a mass effect with midline shift. What is the most likely diagnosis? [B5 Q32]
(a) Ganglioglioma
(b) Calcified arteriovenous malformation
(c) Oligodendroglioma
(d) Pilocytic astrocytoma
(e) Meningioma
Oligodendroglioma
These tumours are seen in young adults and are usually located in the peripheral cerebrum. They typically begin in the hemispheric white matter and grow towards the cortex, are well circumscribed but non-encapsulated, and calcification is a common feature.
A 42-year-old man presents in the Accident & Emergency Department with epileptic seizure. Head CT shows asymmetrical white matter oedema in the left parietal region with a mass effect. Post-contrast study shows a large, irregular, and peripheral enhancing lesion with a central area of low attenuation. What is the most likely diagnosis? [B5 Q41]
(a) Lymphoma
(b) Metastasis
(c) Glioblastoma multiforme
(d) Toxoplasmosis
(e) Cerebral abscess
Glioblastoma multiforme
These tumours are typically inhomogeneous on CT and MRI, showing irregular areas of peripheral enhancement. Tumour necrosis is a hallmark of glioblastoma multiforme.
A 35-year-old man attends the accident and emergency department complaining of episodic lower back pain radiating down the legs. History and clinical examination also suggest pelvic sphincter dysfunction. MRI shows a spinal cord mass located at the conus medullaris. The mass is isointense on T1 and hyperintense on T2. It demonstrates contrast enhancement. The most likely diagnosis is: [B2 Q25]
a. Astrocytoma
b. Intradural lipoma
c. Haemangioblastoma
d. Myxopapillary ependymoma
e. Ganglioglioma
Myxopapillary ependymoma
This variant of ependymoma is the most common neoplasm of the conus medullaris, originating from ependymal glia of the filum terminale. It typically presents around age 35 and is more common in men. T1-weighted imaging shows an isointense or occasionally hyperintense mass, hyperintense on T2, and almost always shows enhancement post-contrast. Intradural lipomas are hyperintense on T1 and do not enhance. Haemangioblastomas can also show high signal on T1 and are highly vascular. Gangliogliomas are more common at the cervical and thoracic levels.
A 9-year-old male presents to the paediatric A&E department with a history of increasing drowsiness over the last 24 hours. Neurological assessment reveals that there is absence of upward gaze. A CT brain is requested which reveals hydrocephalus, with marked dilatation of the lateral and third ventricles. The fourth ventricle is unremarkable and there is obliteration of the ambient cistern due to mass effect from a hyperdense mass noted posterior to the third ventricle. This mass has some central areas of calcification. There is no evidence of haemorrhage. An MRI is carried out following insertion of a shunt to decompress the ventricles. On sagittal sequences a lesion is located between the splenium of the corpus callosum and the tectal plate, which exerts mass effect. This lesion is of intermediate signal intensity on both T1WI and T2WI and displays avid contrast enhancement. Enhancing meningeal lesions are also noted in the spinal cord, indicating seeding. What is the most likely diagnosis? [B1 Q1]
A. Pituitary teratoma.
B. Meningioma.
C. Pineoblastoma.
D. Germinoma.
E. Pineal cyst.
Germinoma.
The lesion is described in the pineal region, so the differential of pineal tumours should be considered. These are differentiated between germ cell tumours and pineal cell tumours. Germinomas account for 40% of all pineal region masses and are much more frequent in males than females. They are also the most common germ cell tumour, with teratomas and choriocarcinoma having different imaging characteristics. The main differential in this case is between pineoblastoma and germinoma, as both occur in patients of this age group and both are hyperdense on CT. Imaging features described to help differentiate are the avid enhancement, which is more characteristic of germinomas, but can occur in either. Central calcification is seen commonly in germinomas, but is uncommon in pineoblastomas, and when it occurs is often peripheral, giving the impression of an ‘exploded’ pineal gland. Subarachnoid seeding is seen in both tumours and if present CSF sampling can yield a tissue diagnosis. Pineocytomas occur in an older age group and infrequently cause subarachnoid seeding.
A 20-year-old male presents with the inability to gaze upwards. CT brain shows moderate hydrocephalus and a rounded mass adjacent to the tectal plate. The mass demonstrates marked homogeneous enhancement and is not calcified. MRI confirms a well-circumscribed, relatively homogeneous mass that is isointense to grey matter on T2-weighted imaging. The mass is hyperintense on contrast-enhanced T1-weighted imaging. What is the most likely diagnosis? [B2 Q1]
a. Germinoma
b. Teratoma
c. Pineoblastoma
d. Pineocytoma
e. Benign pineal cyst
Germinoma
Germinomas are germ-cell tumours arising from primordial germ cells. They frequently occur in the midline, mostly in the pineal region but also in the suprasellar region. In men, 80% of pineal masses are germ-cell tumours, in contrast to 50% in women. They tend to occur in children or young adults (10–25 years old). Symptoms depend on the location, but the case describes Parinaud syndrome – paralysis of upward gaze due to compression of the mesencephalic tectum. Germinomas may also cause hydrocephalus by compression of the aqueduct of Sylvius, thus patients may present with signs and symptoms of raised intracranial pressure. Germinomas are a known cause of precocious puberty in children under the age of ten years. They are malignant tumours and may show CSF seeding, making cytological diagnosis possible with lumbar puncture. They are, however, very radiosensitive and show excellent survival rates. Pineal teratomas tend to be heterogeneous masses containing fat and calcifications. Pineoblastoma is a highly malignant tumour which is more common in children and usually has poor tumour margins.
A 30-year-old man presents with an orbital frontal headache and visual disturbance. CT head shows a large mass arising from the region of the spheno-occiput and extending into the pontine cistern and towards the hypothalamus. The mass contains amorphous calcification and is seen to cause bone destruction. There is no reactive bone sclerosis. On MRI the mass exhibits mixed heterogeneous signal and a soap bubble appearance. The solid components show marked contrast enhancement. What is the most likely diagnosis? [B2 Q36]
a. Meningioma
b. Metastasis
c. Chordoma
d. Plasmacytoma
e. Sphenoid sinus cyst
Chordoma
Chordomas originate from malignant transformation of notochordal cells. They are typically located in the sacrum (50%), clivus (35%) and vertebrae (15%). They may rarely be found in the mandible, maxilla and scapula. Spheno-occipital chordomas typically affect males and females in equal incidence and the average age ranges from 20 to 40 years. Most of them demonstrate bone destruction and amorphous calcification. The solid components show variable but often marked contrast enhancement and MRI may show a ‘soap-bubble’ appearance. Bone sclerosis is rare. The description given is that of a malignant process. Meningiomas typically affect older females and only rarely (<1%) arise from the clivus. Plasmacytomas tend to occur in an older age group, are more common in the thoracic/lumbar spine and are often osteolytic and grossly expansile lesions. Metastasis is a possibility, although from the description given – particularly the calcification and amount of tumour extension – chordoma remains the most likely diagnosis.
A 35-year-old male presents with ataxia and nystagmus. Blood tests reveal polycythaemia. CT head demonstrates a mass predominantly of CSF density in the posterior fossa. Subsequent MRI shows a largely cystic mass with an enhancing mural nodule. There is surrounding oedema but no calcification. The most likely diagnosis is: [B2 Q5]
a. Metastasis
b. Pilocytic astrocytoma
c. Haemangioblastoma
d. Choroid cyst
e. Ependymoma
Haemangioblastoma
Haemangioblastoma is the most common primary intra-axial, infratentorial tumour in adults. They are benign autosomal dominant tumours of vascular origin. Approximately 20% occur with von Hippel–Lindau disease. Other associations include phaeochromocytomas, syringomyelia and spinal cord haemangioblastomas. About 20% of tumours cause polycythaemia. Typical CT and MRI appearances are of a largely cystic mass with an enhancing mural nodule. Oedema may be absent or extensive, but calcification is rare. Prognosis is 85% post-surgical five-year survival rate. Infratentorial pilocytic astrocytomas may have very similar appearances to haemangioblastomas but some differences exist that can help differentiate the two. Pilocytic astrocytomas predominantly occur in children and young adults, are generally larger (>5 cm) than haemangioblastomas, may contain calcifications and are not associated with polycythaemia.
A 22-year-old man with von Hippel–Lindau syndrome presents with headaches, vomiting and ataxia. CT of the brain demonstrates an abnormality in the posterior fossa. What are the most likely findings? [B4 Q27]
a. cystic lesion with an enhancing mural nodule
b. multiple ring-enhancing lesions with surrounding oedema
c. gyriform cortical calcifications
d. multiple calcified subependymal nodules
e. hypodense, ill-defined mass with central necrosis and marked surrounding oedema
Cystic lesion with an enhancing mural nodule
Haemangioblastomas are the most recognized manifestation of von Hippel–Lindau syndrome. They usually occur in the cerebellum and may be multiple in up to 15% of cases. Patients also commonly develop renal cell carcinoma, and difficulty may occasionally arise in distinguishing metastases from multiple haemangioblastomas. Typical features are of a cystic mass in the hemisphere or vermis, with an enhancing mural nodule, though entirely solid lesions may also occur. Calcification is not a feature. Gyriform cortical calcifications are a feature of Sturge–Weber syndrome, and usually occur in the temporoparieto-occipital region. Calcified subependymal nodules (hamartomas) are a feature of tuberous sclerosis. A hypodense, ill-defined mass with central necrosis and marked surrounding oedema is a classic appearance of a glioblastoma multiforme.
A 40-year-old man presents with gradually worsening symptoms of ataxia, nausea and vomiting. CT of the head shows a 2 cm cystic lesion in the cerebellum with an enhancing mural nodule. What is the most likely diagnosis? [B5 Q10]
(a) Cerebellar haemangioblastoma
(b) Metastasis
(c) Cystic astrocytoma
(d) Arachnoid cyst
(e) Medulloblastoma
Cerebellar haemangioblastoma
These CT features are typical of this lesion.
Metastases usually have ring-like enhancement or homogenous enhancement rather than having a mural nodule. Cystic astrocytomas are usually > 5 cm, show calcifications, thick walled and have no enhancing mural nodule. An arachnoid cyst is a possibility if no enhancing nodule seen. Medulloblastoma is uncommon in adults and is usually a solid tumour with homogenous enhancement.
A 35-year-old man presents with headache and ataxia. CT of the brain shows a 6 cm cystic lesion in the right cerebellar hemisphere with a small enhancing nodule at the margin of the cyst. The most likely diagnosis is? [B5 Q25]
(a) Arachnoid cyst
(b) Necrotic metastasis
(c) Haemangioblastoma
(d) Juvenile pilocytic astrocytoma
(e) Cysticercosis
Haemangioblastoma
This is the most common posterior fossa tumour in adults after metastases. They are usually seen in the cerebellum and there is an association with von Hippel–Lindau disease. CT appearances are typically with a large hypodense cyst and an enhancing mural nodule. The cyst wall does not usually enhance. On MRI, flow voids may be seen representing draining vessels adjacent to the nodule.
Juvenile pilocytic astrocytoma is seen in young age and is not associated with feeding vessels. Metastases are usually multiple in older people.
A 40-year-old female is investigated for worsening headaches. CT shows a well-defined hyperdense globular lesion within the trigone of the left lateral ventricle. There is intense contrast enhancement. The most likely diagnosis is: [B2 Q2] {will repeat again in Meningioma Section}
a. Choroid cyst
b. Ependymoma
c. Colloid cyst
d. Meningioma
e. Neurocytoma
Meningioma
It is rare for meningiomas to occur intraventricularly (2–5% of all meningiomas) but they are the most common trigonal intraventricular mass in adulthood. They tend to occur in 40-year-old females.
When on call you are asked to perform a CT head scan for a 17-year-old male who presents with seizures. He is unable to provide a history. A look on the computer system shows that he has had previous regular abdominal ultrasounds and an echocardiogram as a child. Brain CT shows a hypodense, well-demarcated, rounded mass in the region of the foramen of Monro. It is partially calcified, and it demonstrates uniform enhancement. What is the most likely diagnosis? [B2 Q4] {will repeat again in periventricular tumour section}
a. Colloid cyst
b. Giant cell astrocytoma
c. Metastasis
d. Lymphoma
e. Haemangioblastoma
Giant cell astrocytoma
The main differential for a mass at the foramen of Monro is between a colloid cyst and a subependymal giant cell astrocytoma. The latter is associated with tuberous sclerosis (TS). Renal involvement is also relatively common in TS and patients regularly have surveillance renal ultrasounds. The echocardiogram was performed to assess for cardiomyopathy/rhabdomyoma.
A 30-year-old female complains of increasing headaches, episodic vomiting, and drowsiness. Fundoscopy reveals papilledema. Non-contrast CT of the head demonstrates hydrocephalus and a globular lesion within the lateral ventricle. There are several small internal foci of calcification. MR shows the mass to be attached to the septum pallucidum. It is isointense to grey matter on T1 and T2. It densely enhances after intravenous gadolinium. What is the most likely diagnosis? [B2 Q23]
a. Ependymoma
b. Subependymoma
c. Central neurocytoma
d. Heterotopic grey matter
e. Meningioma
Central neurocytoma
Central neurocytoma is an intraventricular WHO grade II neuroepithelial tumour with neuronal differentiation. This rare neoplasm tends to occur between the ages of 20–40 years. Patients typically present with symptoms and signs of hydrocephalus.
Imaging typically demonstrates a globular lesion attached to the septum pellucidum. Calcification is considered characteristic; however, it may be absent in approximately half the cases. On MRI, the lesions are usually isointense to grey matter and show dense contrast enhancement. It is extremely uncommon to see peri-tumoural oedema.
Which of the following is an extra-axial posterior fossa tumour in adults? [B3 Q10]
A. Choroid plexus papilloma
B. Metastasis
C. Haemangioblastoma
D. Lymphoma
E. Glioma
Choroid plexus papilloma
Other extra-axial posterior fossa masses include acoustic neuroma, meningioma, chordoma and epidermoid.
Which is the most prevalent intraventricular tumour? [B3 Q11]
A. Astrocytoma
B. Meningioma
C. Choroid plexus papilloma
D. Dermoid
E. Subependymoma
A
Ependymomas followed by astrocytoma are the most common.
Which of the following best describes imaging changes in a colloid cyst? [B3 Q18]
A. Typically hypodense on non-contrast CT
B. Appears high SI on T1
C. Appears low SI on T2
D. Commonly widens septum pellucidi
E. Most commonly causes symmetrical enlargement of lateral ventricles
B
Protein content/paramagnetic effect of Magnesium Mg2+/ Calcium Ca2+/ Iron Fe, in a cyst cause increased T1 and T2 SI. Colloid cysts appear iso/hyperdense on NCCT. They can occasionally widen septum pellucidum and cause asymmetrical enlargement of the lateral ventricles.
A 27-year-old, previously well man gives a history of positional headaches. He undergoes unenhanced CT of the brain, which demonstrates a well-defined, hyperdense, spherical lesion in the anterosuperior portion of the third ventricle, with asymmetrical enlargement of the lateral ventricles. What is the most likely diagnosis? [B4 Q58]
a. meningioma
b. colloid cyst
c. ependymoma
d. subependymal giant cell astrocytoma
e. choroid plexus papilloma
Colloid Cyst
Colloid cysts arise from the inferior aspect of the septum pellucidum and protrude into the anterior aspect of the third ventricle, where they may cause positional headaches and hydrocephalus due to transient obstruction at the foramen of Monro. They usually contain mucinous fluid, desquamated cells and proteinaceous debris, making them hyperdense on CT. On MR scan, the high protein content, as well as the paramagnetic effect of magnesium, copper and iron in the cyst, results in high signal intensity on T1W and T2W sequences in 60% of cases. Meningiomas and ependymomas may also appear hyperdense, but location within the third ventricle is uncommon for both. Subependymal giant cell astrocytoma is a benign tumour occurring in the region of the foramen of Monro, which may cause obstruction, but it is usually hypodense and is nearly always seen in association with tuberous sclerosis. Choroid plexus papilloma predominantly occurs in children under 5 years of age, and location in the third ventricle is unusual.
A 21-year-old man presents with acute headache. He undergoes CT of the brain, which demonstrates a well-circumscribed, lobulated, partially cystic, calcified mass in the frontal horn of the left lateral ventricle, attached to the septum pellucidum. There is acute blood seen layering in the left lateral ventricle and mild hydrocephalus. What is the most likely diagnosis? [B4 Q99]
a. choroid plexus papilloma
b. colloid cyst
c. intraventricular oligodendroglioma
d. central neurocytoma
e. subependymoma
Central neurocytoma
Central neurocytomas (also known as intraventricular neurocytomas) are benign tumours of the lateral and third ventricles usually presenting in adults aged 20–40 years. They frequently calcify (69%) and contain cystic spaces. Attachment to the septum pellucidum is a characteristic feature. Lesions appear isointense to grey matter on all MR sequences and show mild-to-moderate contrast enhancement. Central neurocytomas were previously frequently mistaken for intraventricular oligodendrogliomas, which have very similar imaging features but are actually quite rare. In addition, neurocytomas undergo haemorrhage into the tumour or ventricle more frequently, helping to distinguish the two. Colloid cysts arise within the third ventricle, and rarely calcify. Subependymomas may arise in the lateral ventricles with an attachment to the septum pellucidum, and cyst formation and calcification may be seen in large tumours. However, most occur in patients over 40 years of age. Choroid plexus papillomas generally occur in children under 5 years of age.
A 37-year-old man presented with a history of intermittent headaches. Unenhanced CT scan of the head demonstrates a 1 cm, dense, round lesion in the region of the interventricular foramen. Mild hydrocephalus was also seen. On MRI, the lesion returns high signal on T1 and T2 sequences. The most likely diagnosis of this lesion is? [B5 Q8]
(a) Meningioma
(b) Ependymoma of the 3rd ventricle
(c) Colloid cyst
(d) Dermoid cyst
(e) Arachnoid cyst
Colloid cyst
Colloid cysts are seen in the region of the interventricular foramen and cause positional and intermittent obstruction, leading to hydrocephalus and headaches. The lesion is dense on CT and high signal on T1 and T2 sequences as they commonly contain large protein molecules and the paramagnetic effect of iron and copper in the cyst.
Dermoid contains fat whereas arachnoid cysts show CSF features on imaging.
Meningioma is usually low signal on T1 and high signal on T2.
A 30-year-old male patient is referred from ENT for an MRI with a history of tinnitus and slight hearing loss on the left side. A lesion is noted in the left cerebellopontine angle. This extends along the nerve and expands the internal auditory canal. A separate nerve is noted to enter the anterior superior portion of the internal auditory canal. The lesion is isointense to the pons on all pulse sequences. The lesion makes an acute angle with the petrous bone. There is no evidence of a dural tail following enhancement. What is the most likely cause? [B1 Q15]
A. Meningioma.
B. Facial nerve schwannoma.
C. Vestibular nerve schwannoma.
D. Epidermoid.
E. Arachnoid cyst.
C. Vestibular nerve schwannoma
This accounts for 75% of CPA tumors. The acute angle with the petrous bone and expansion of the internal auditory canal are characteristic features. The absence of a dural tail further supports this diagnosis. Facial nerve schwannomas are rare in this location and typically cannot be separated from the mass on imaging.
A 42-year-old woman has a CT scan of the brain performed for the investigation of headache. A lesion with density like cerebrospinal fluid is noted at the right cerebellopontine angle. A subsequent MRI scan shows that the lesion is well-defined, of decreased signal on T1WI and increased signal on T2WI. You think the lesion is most likely an epidermoid cyst but wish to exclude an arachnoid cyst. Which of the following further MRI sequences will be most helpful in achieving this aim? [B1 Q25]
A. Proton density.
B. STIR.
C. T1 with fat suppression.
D. FLAIR.
E. T1 post gadolinium.
D. FLAIR
On FLAIR sequences, epidermoid cysts show mixed iso- to hyperintense signal with poor demarcation, while arachnoid cysts have their signal suppressed, resembling CSF. Diffusion-weighted imaging can also help distinguish them, as epidermoid cysts show very high signal due to restricted diffusion.
A 54-year-old man presents with hearing loss in the left ear, which is of the sensori-neural type. He undergoes an MRI scan of the internal auditory meati and subsequently a full MRI scan of the brain. A large extra-axial mass lesion is identified at the left cerebellopontine angle. Which of the following features on MR imaging will be most helpful in indicating that this is probably a large vestibular schwannoma, rather than a meningioma? [B1 Q74]
A. Enhancing dura adjacent to the mass.
B. Erosion of the adjacent porus acousticus.
C. Tumour within the adjacent internal auditory meatus.
D. Intense enhancement within the mass.
E. Hyperostosis in the adjacent petrous temporal ridge.
B. Erosion of the adjacent porus acousticus
Vestibular schwannomas typically erode the posterior edge of the porus acousticus as they grow, whereas meningiomas usually do not cause such erosion. This distinction is crucial in identifying the nature of the lesion.
A 13-year-old boy is investigated for chronic headache and visual disturbance. CT shows a well-defined mass in the left middle cranial fossa. It is isodense to CSF. There are no calcifications, no surrounding edema, and no contrast enhancement. There is erosion of the underlying calvarium. You suspect this is an arachnoid cyst, but your consultant suggests the possibility of an epidermoid cyst. What MR imaging sequence would best differentiate the two? [B2 Q26]
a. Diffusion-weighted MR imaging
b. Gadolinium-enhanced T1-weighted imaging
c. Proton density imaging
d. MR spectroscopy
e. Perfusion-weighted MR imaging
A. Diffusion-weighted MR imaging
Epidermoid cysts appear bright on diffusion-weighted imaging due to restricted diffusion, while arachnoid cysts appear dark. This sequence is particularly useful for distinguishing between the two cyst types when they are isodense to CSF on CT and standard MRI.
Which of the following favours an arachnoid rather than an epidermoid cyst? [B3 Q15]
A. Hyperdense to CSF on CT
B. Smooth margin
C. Encased vessels
D. Deviates from CSF on proton density
E. Restricted diffusion
B. Smooth margin
Smooth margins are characteristic of arachnoid cysts, while epidermoid cysts tend to have irregular margins and can encase vessels.
Which is the most common location for epidermoid in the Central Nervous System (CNS)? [B3 Q19]
A. Cerebellar pontine angle (CPA)
B. Suprasellar region
C. Perimesencephalic cisterns
D. Ventricles
E. Skull vault
A. Cerebellar pontine angle (CPA)
Epidermoid cysts are most commonly found in the CPA, accounting for 40% of cases and 5% of CPA tumors.
A 2cm mass is seen on MR at the left CPA with uniform enhancement and high SI on T2 and dural tail. What is the diagnosis? [B3 Q33]
A. Meningioma
B. Vestibular schwannoma
C. Epidermoid
D. Metastatic deposit
E. Glomus tumour
A. Meningioma
The presence of a dural tail and broad-based attachment to the petrous bone are characteristic of meningiomas. Uniform enhancement and less bright T2 signal further support this diagnosis
35-year-old woman presents with progressive deafness and tinnitus in the left ear. She undergoes MRI, which demonstrates a 2 cm mass at the left cerebellopontine angle. Which of the following features would favour a diagnosis of meningioma rather than vestibular schwannoma? [B4 Q18]
a. acute angle with the petrous bone
b. hyperintensity on T2W images
c. expansion of the internal auditory canal
d. presence of a dural tail
e. internal cystic degeneration and haemorrhage
Presence of a dural tail
The most common causes of a cerebellopontine angle mass are vestibular schwannoma (also called acoustic neuroma) (75%), meningioma (10%) and epidermoid cyst (5%). Features suggestive of a meningioma include a dural tail (thickening of enhancing adjacent dura resembling a tail extending from the mass), adjacent hyperostosis and an obtuse angle with the petrous bone (vestibular schwannomas make an acute angle). Distinguishing features of schwannomas include extension into the internal auditory canal, causing expansion of the canal and flaring of the porus acousticus (bony opening of the internal auditory canal). Meningiomas may show a small tongue of extension into the canal but usually no expansion. Schwannomas undergo cystic degeneration and haemorrhage more commonly than meningiomas (particularly larger lesions), and may show very high signal on T2W images, which is unusual for a meningioma.
A 23-year-old man presents with acute headache. Unenhanced CT of the brain demonstrates a heterogeneous mass at the inferior cerebellar vermis. It is predominantly of fat attenuation with areas of calcification and does not enhance following administration of intravenous contrast. Multiple droplets of fat attenuation are noted throughout the subarachnoid space. What is the most likely diagnosis? [B4 Q72]
a. lipoma
b. arachnoid cyst
c. dermoid
d. epidermoid
e. teratoma
Dermoid
Epidermoids and dermoids are congenital lesions resulting from inclusion of ectodermal elements during closure of the neural tube. Both have a squamous epithelial lining and produce keratin, but dermoids contain both ectodermal and mesodermal elements (hair follicles, sweat and sebaceous glands), while epidermoids contain only ectodermal elements. Epidermoids are lobulated masses, usually located off the midline, which compress adjacent structures such as cranial nerves, and tend to have appearances on CT and MR scan following that of CSF. Dermoids are usually midline in location and cause symptoms by obstruction of CSF pathways or by rupture and leakage of fat contents, causing chemical meningitis. They have appearances on CT and MR scan following that of fat and are often heterogeneous with areas of calcification and other soft-tissue components. This heterogeneity helps to distinguish a dermoid from a lipoma. Arachnoid cysts are of CSF density, but may be distinguished from epidermoid on DWI, where they do not show evidence of water restriction. Teratomas are composed of ectodermal, mesodermal and endodermal elements, and also appear heterogeneous with areas of fat, calcification and cystic components, but these lesions occur most commonly in the pineal and suprasellar regions.
A 40-year-old teacher presents with a history of hearing loss in the left ear. Gadolinium-enhanced MRI shows a non-enhancing lesion in the left cerebellopontine angle (CPA). The lesion is isointense to CSF on T1 and T2 sequences. On FLAIR imaging, the lesion shows incomplete attenuation of fluid signal and on diffusion-weighted imaging it returns a bright signal. The most likely diagnosis is? [B5 Q11]
(a) Arachnoid cyst in the left CPA
(b) Schwannoma in the left CPA
(c) Epidermoid cyst in the left CPA
(d) Lipoma in the left CPA
(e) Cystic meningioma in the left CPA
(c) Epidermoid cyst in the left CPA
This resembles CSF on non-enhanced CT. On MRI, the lesion is isointense, or slightly hyperintense to CSF. Incomplete attenuation on FLAIR and high signal on diffusion (suggesting restricted diffusion) concludes the diagnosis. Non enhancement is the rule.
A 50-year-old woman complains of tinnitus, headaches, and hearing loss. MRI shows a heterogenous, well-defined mass in the left cerebellopontine angle producing a local mass effect. The lesion returns low signal on T1, heterogenous high signal on T2 and heterogeneously enhances with gadolinium. The most likely diagnosis is? [B5 Q20]
(a) Schwannoma
(b) Meningioma
(c) Epidermoid
(d) Arachnoid cyst
(e) Metastasis
(a) Schwannoma
Vestibular schwannomas are the most common cerebellopontine angle tumours with typical imaging features as given above. These are typically benign, slow growing tumours from Schwann cells which envelop and myelinate cranial, spinal and peripheral nerves. In the skull, they most commonly arise in the cerebellopontine angle, from the vestibular portion of the 8th cranial nerve. The main differential diagnosis is meningioma, which can also grow from the 8th cranial nerve. A schwannoma is hyperintense on T2 while a meningioma is isohypointense. Also, a meningioma forms an obtuse angle with the petrous bone while a schwannoma forms an acute angle. Finally, a meningioma may have a dural tail, which is absent in a schwannoma.
A 18-year-old man was admitted after a road traffic accident. CT of the head shows an incidental lesion in the right cerebello-pontine angle. MRI shows a 4 cm homogenous lesion in the right cerebellopontine angle which is high signal on T2, intermediate on T1 and without restricted diffusion. No gadolinium enhancement seen. What is the most likely diagnosis? [B5 Q31]
(a) Arachnoid cyst
(b) Acoustic neuroma
(c) Epidermoid cyst
(d) Lipoma
(e) Necrotic metastasis
(a) Arachnoid cyst
An arachnoid cyst returns signal characteristics of cerebrospinal fluid. High on T2, intermediate on T1 and no restriction on diffusion-weighted images.
A 40-year-old man presents with unilateral sensorineural hearing loss. MRI shows a well-defined mass in the left cerebellopontine angle. The lesion returns high signal on T1, T2 and FLAIR sequences. On fat saturated T1 with contrast, the lesion returns low signal and no contrast enhancement is seen. What is the most likely diagnosis? [B5 Q37]
(a) Acoustic schwannoma with haemorrhage
(b) Lipoma
(c) Epidermoid cyst
(d) Giant aneurysm
(e) Arachnoid cyst
Cerebellopontine angle lipoma
The lesion has characteristics of fat on all sequences (high signal on T1 and T2 with loss of signal on fat-suppression imaging).
A 12-year-old male with a history of gelastic seizures is referred for MRI of the brain. Which of the following statements regarding hamartomas of the tuber cinereum is true? [B1 Q41]
A. No change in size, shape, or signal intensity on follow-up MRI.
B. Demonstrate homogenous contrast enhancement.
C. Calcification is a common finding.
D. Hyperintense on T1WI and T2WI, and hypointense on fat suppressed sequences.
E. Located in the sella turcica.
No change in size, shape, or signal intensity on follow-up MRI.
Hypothalamic hamartomas are developmental malformations located in the tuber cinereum of the hypothalamus. The typical patient is male, in the first or second decade of life, presenting with precocious puberty or gelastic seizures. On MRI, they appear as well-defined pedunculated or sessile lesions that are iso/mildly hypointense on T1WI and iso/hyperintense on T2WI, with no contrast enhancement or calcification. Lack of interval change strongly supports the diagnosis.
A 45-year-old female undergoes an MRI of the pituitary that demonstrates a kidney-shaped lesion located centrally in the pituitary fossa in the axial plane. It is hyperintense on T1WI and hypointense on T2WI. There is no enhancement following gadolinium administration and no fluid–fluid level. What is the diagnosis? [B1 Q53]
A. Rathke cleft cyst.
B. Craniopharyngioma.
C. Cholesterol granuloma.
D. Haemorrhagic adenoma.
E. Lipoma.
Rathke cleft cyst.
These are benign cystic lesions of the pituitary fossa derived from the Rathke pouch. They are usually asymptomatic. Rathke cleft cysts are in the midline between the anterior and posterior pituitary lobes and have a characteristic kidney shape on axial images. They are homogenously hyperintense on T1WI, due to high protein concentration, and hypointense on T2WI, due to low intracystic water content. They do not enhance following contrast administration. Absence of a fluid–fluid level is helpful in differentiating Rathke cleft cyst from hemorrhagic adenoma. Acute haemorrhage has heterogenous signal on T2WI and may demonstrate thin peripheral enhancement on T1WI. Craniopharyngiomas have variable solid, cystic, and calcified components. They demonstrate heterogenous enhancement following contrast. They may be intrasellar or suprasellar. A pseudo-fluid–fluid level may occasionally be seen in craniopharyngioma. Lipoma and cholesterol granuloma are hyperintense on both T1WI and T2WI.
A 32-year-old female is referred to neurology complaining of visual disturbance and headache. She is 4 months postpartum. On examination a bitemporal hemianopia is noted. Hormonal testing reveals hypoadrenalism and hypothyroidism. A dedicated MRI of her pituitary gland is requested. Which of the following features is suggestive of autoimmune hypophysitis over pituitary adenoma? [B1 Q75]
A. Asymmetric pituitary enlargement.
B. Heterogenous gadolinium enhancement.
C. Loss of the posterior pituitary bright spot.
D. Sphenoid sinus mucosal thickening.
E. Age >30.
Loss of the posterior pituitary bright spot.
Autoimmune hypophysitis (AH) and non-secreting pituitary adenomas can only be differentiated with certainty on histology. As a result, approximately 40% of patients with AH are misdiagnosed as having pituitary macroadenoma and undergo unnecessary surgery. Hormone production is compromised in both conditions, although a history of infertility is common with adenomas, whereas patients with AH typically achieve spontaneous pregnancy. To develop a scoring system to differentiate the two conditions, a recent study found that features significantly associated with AH over adenoma were age <30, relation to pregnancy, homogenous gadolinium enhancement, loss of the posterior pituitary bright spot, and enlarged stalk. Features consistent with adenoma were asymmetrically enlarged pituitary, size >6 cm³, and associated sinus mucosal thickening. The normal posterior pituitary gland is bright on T1WI due to the rich content of vasopressin neurosecretory granules. This is frequently lost in AH due to direct autoimmune involvement of the neurohypophysis, whereas it is conserved in most adenomas, even when displaced by large tumour size.
A 40-year-old female presents with bitemporal hemianopia. CT brain shows a large, slightly hyperdense suprasellar lesion. The mass contains several lucent foci and there is bone erosion of the sella floor. There is enhancement post-contrast. T1-weighted MR imaging shows a predominantly isointense mass causing sella expansion and compression of the optic chiasm. The mass contains foci of low and high signal intensity. What is the most likely diagnosis? [B2 Q3]
a. Craniopharyngioma
b. Meningioma
c. Rathke’s cleft cyst
d. Giant internal carotid aneurysm
e. Pituitary adenoma
Pituitary adenoma
Pituitary adenomas are divided into microadenomas (<1 cm) and macroadenomas (>1 cm). Macroadenomas may present with endocrine dysfunction but are generally less active than microadenomas. Thus, macroadenomas often present with symptoms of mass effect on the optic chiasm, or if there is lateral extension into the cavernous sinuses, patients may present with other local cranial nerve palsies (III, IV, VI). The differential diagnosis of a suprasellar mass includes (‘SATCHMO’): Suprasellar extension of pituitary adenoma/sarcoid; Aneurysm/arachnoid cyst; TB/teratoma (other germ-cell tumours); Craniopharyngioma; Hypothalamic glioma or hamartoma; Meningioma/metastases (especially breast); and Optic/chiasmatic glioma. In this case, the sellar is widened and the floor is eroded suggesting the mass arises from the pituitary itself. Low-density/low-intensity regions on CT/T1 MRI correspond to necrotic areas and high-signal foci on T1 MRI (found relatively frequently) represent areas of recent haemorrhage.
Which of the following features favour Rathke’s cleft cyst rather than craniopharyngioma? [B3 Q1]
A. Absence of calcification
B. Cystic element on MR
C. Involvement of suprasellar and sellar regions
D. Enhancement of the wall
E. High signal intensity on T1
Absence of calcification
Rathkes cleft cysts do not calcify. They affect women to men in a 2:1 ratio and adults from 40-60 years of age. They cause variable MR appearances depending on protein content of cyst. They can rarely show enhancement.
What are the typical appearances of a pituitary microadenoma on early, contrast enhanced T1W MR images? [B4 Q13]
a. focus of hypo-intensity within normal enhancing gland
b. focus of enhancement within normal, non-enhancing gland
c. lesion and normal gland enhance similarly
d. hyper-enhancing focus within normal, mildly enhancing gland
e. not usually visualized on this sequence
Focus of hypo-intensity within normal enhancing gland
Pituitary microadenomas are typically hypointense compared with the normal gland on unenhanced T1W images, and the diagnosis can usually be made without contrast. Following contrast, the microadenoma does not initially enhance, and maximal contrast between enhancing normal gland and pituitary tumour is seen on dynamic images obtained within the first minute. Contrast enhancement may therefore be useful in identifying lesions that are not obviously hypointense on the unenhanced images. However, contrast enhancement of the tumour relative to the normal gland may be seen on delayed (>20 min) images.
A 46-year-old man presents with headaches and visual disturbance and is found to have bitemporal hemianopia on visual field testing. CT of the brain demonstrates a multilobulated, heterogeneous, suprasellar mass containing cystic areas and rim-like calcification. Enhancement of the solid component is observed following intravenous contrast. What is the most likely diagnosis? [B4 Q57]
a. epidermoid
b. craniopharyngioma
c. pituitary macroadenoma
d. meningioma
e. Rathke’s cleft cyst
Craniopharyngioma
Craniopharyngiomas are the most common suprasellar mass, predominantly occurring in the first and second decades, but with a second peak in the fifth decade. Presenting symptoms include headache secondary to hydrocephalus, bitemporal hemianopia (compression of the optic chiasm) and diabetes insipidus (compression of the pituitary gland). Typical imaging features are calcifiication, cyst formation and enhancement that may be solid or nodular. Meningiomas may arise in the suprasellar region and commonly demonstrate calcifiication but are generally not cystic. Epidermoids may occasionally demonstrate rim calcifiication but rarely enhance following intravenous contrast. Pituitary macroadenomas may undergo haemorrhage, resulting in heterogeneity that can cause confusion with craniopharyngioma, but this typically occurs in adolescence. Calcifiication in macroadenomas is infrequent. Rathke’s cleft cysts are thin-walled, benign cysts arising in the anterior sellar or suprasellar region. They show no contrast enhancement and rarely calcify.
A 1-year-old child presents with precocious puberty and MRI shows a suprasellar mass attached to the mamillary bodies with a thin stalk. The most likely cause is? [B5 Q4]
(a) Hypothalamic hamartoma
(b) Craniopharyngioma
(c) Hypothalamic glioma
(d) Kallman syndrome
(e) Pituitary adenoma
Hypothalamic hamartoma
Also called hamartoma of the tuber cinereum, this is seen in children less than 2 years of age. Precocious puberty is due to luteinising releasing hormone secretion. Craniopharyngioma presents with growth failure and visual field defects. Kallmann syndrome presents with hypogonadism in later age. Pituitary adenomas are seen in girls (9–13 years of age) and are usually prolactin or adrenocorticotropic hormone secreting lesions.
A 21-year-old boy with neurofibromatosis type 1 complains of visual difficulties. MRI shows abnormal enlargement of the optic chiasm and intense and homogenous enhancement with gadolinium. The abnormality extends into the left optic tract. What is the most likely diagnosis? [B5 Q33] {will repeat again in neuro-cutaneous syndrome section}
(a) Craniopharyngioma
(b) Lymphoma
(c) Neurosarcoidosis
(d) Chiasmal glioma
(e) Tuberculosis
Chiasmal glioma
Chiasmal glioma is associated with NF1 in 15-25% cases. Craniopharyngioma tends to displace the chiasm rather than enlarge it. It is often cystic and may show calcification. Lymphoma is more likely to involve peripheral nerves. Tuberculosis may involve the chiasm but in the setting of basal meningitis, leptomeningeal enhancement and multiple cranial neuropathies.
A 9-year-old male presents to the paediatric A&E department with a history of increasing drowsiness over the last 24 hours. Neurological assessment reveals that there is absence of upward gaze. A CT brain is requested which reveals hydrocephalus, with marked dilatation of the lateral and third ventricles. The fourth ventricle is unremarkable and there is obliteration of the ambient cistern due to mass effect from a hyperdense mass noted posterior to the third ventricle. This mass has some central areas of calcification. There is no evidence of haemorrhage. An MRI is carried out following insertion of a shunt to decompress the ventricles. On sagittal sequences, a lesion is located between the splenium of the corpus callosum and the tectal plate, which exerts mass effect. This lesion is of intermediate signal intensity on both T1WI and T2WI and displays avid contrast enhancement. Enhancing meningeal lesions are also noted in the spinal cord, indicating seeding. What is the most likely diagnosis? [B1 Q1]
A. Pituitary teratoma.
B. Meningioma.
C. Pineoblastoma.
D. Germinoma.
E. Pineal cyst.
Germinoma.
The lesion is described in the pineal region, so the differential of pineal tumours should be considered. These are differentiated between germ cell tumours and pineal cell tumours. Germinomas account for 40% of all pineal region masses and are much more frequent in males than females. They are also the most common germ cell tumour, with teratomas and choriocarcinoma having different imaging characteristics. The main differential in this case is between pineoblastoma and germinoma, as both occur in patients of this age group and both are hyperdense on CT. Imaging features described to help differentiate are the avid enhancement, which is more characteristic of germinomas, but can occur in either. Central calcification is seen commonly in germinomas, but is uncommon in pineoblastomas, and when it occurs is often peripheral, giving the impression of an ‘exploded’ pineal gland. Subarachnoid seeding is seen in both tumours and if present CSF sampling can yield a tissue diagnosis. Pineocytomas occur in an older age group and infrequently cause subarachnoid seeding.
A 20-year-old male presents with the inability to gaze upwards. CT brain shows moderate hydrocephalus and a rounded mass adjacent to the tectal plate. The mass demonstrates marked homogeneous enhancement and is not calcified. MRI confirms a well-circumscribed, relatively homogeneous mass that is isointense to grey matter on T2-weighted imaging. The mass is hyperintense on contrast-enhanced T1-weighted imaging. What is the most likely diagnosis? [B2 Q1]
a. Germinoma
b. Teratoma
c. Pineoblastoma
d. Pineocytoma
e. Benign pineal cyst
Germinoma.
Germinomas are germ-cell tumours arising from primordial germ cells. They frequently occur in the midline, mostly in the pineal region but also in the suprasellar region. In men, 80% of pineal masses are germ-cell tumours, in contrast to 50% in women. They tend to occur in children or young adults (10–25 years old). Symptoms depend on the location, but the case describes Parinaud syndrome – paralysis of upward gaze due to compression of the mesencephalic tectum. Germinomas may also cause hydrocephalus by compression of the aqueduct of Sylvius, thus patients may present with signs and symptoms of raised intracranial pressure. Germinomas are a known cause of precocious puberty in children under the age of ten years. They are malignant tumours and may show CSF seeding, making cytological diagnosis possible with lumbar puncture. They are, however, very radiosensitive and show excellent survival rates. Pineal teratomas tend to be heterogeneous masses containing fat and calcifications. Pineoblastoma is a highly malignant tumour which is more common in children and usually has poor tumour margins.
A 9-year-old boy presents with precocious puberty and headache. CT of the brain shows an enhancing mass in the pineal region with calcifications. There is moderate hydrocephalus with a dilated lateral and 3rd ventricle. The most likely diagnosis is? [B5 Q21]
(a) Pineal germinoma
(b) Glioma
(c) Medulloblastoma
(d) Meningioma
(e) Metastases
Pineal germinoma (also called pinealoma).
This is the most common pineal tumour and is associated with precocious puberty in children less than 10 years old. The finding of pineal calcification before 10 years with a pineal mass enhancing with contrast is usually diagnostic. Hydrocephalus is secondary to compression of the cerebral aqueduct. Medulloblastoma is a tumour usually in the posterior fossa and presenting in childhood. Suprasellar meningioma does not arise from the pituitary fossa.
A 49-year-old woman with AIDS presents with increasing headache. T1-weighted MR imaging demonstrates a hypointense lesion in the periventricular white matter of the left parietal lobe. The lesion is hypointense on FLAIR sequencing and is seen to cross the splenium of the corpus callosum. There is peripheral enhancement post-contrast injection. The most likely diagnosis is: [B2 Q44]
a. Toxoplasmosis
b. Progressive multifocal leukoencephalopathy
c. Primary CNS lymphoma
d. Cryptococcosis
e. Tuberculosis
Primary CNS lymphoma (PCNSL).
PCNSL is the second most common cause of a CNS mass in patients with AIDS (behind toxoplasmosis). Typical features include periventricular location with subependymal spread and crossing of the corpus callosum. Non-contrast CT may show a hyperdense lesion due to dense cellularity (for this reason it may also be hypointense on T2/FLAIR). There is often a paucity of oedema and frequent ring enhancement due to central necrosis (note this is in contrast with the solid homogeneous enhancement seen with lymphoma in the immunocompetent patient). Cryptococcosis is the most common cause of fungal infection in AIDS patients. CT is frequently normal, and MRI shows low T1 and high T2 signal intensities without enhancement in the lenticulostriate region. Progressive multifocal leukoencephalopathy typically shows bilateral white matter lesions in the periventricular region, centrum semi-ovale or subcortical white matter, which are hypointense on T1 and hyperintense on T2/FLAIR. There is typically no oedema nor mass effect and no contrast enhancement.
A 35-year-old man presents with persistent headaches. CT of the head shows a 3 cm homogenous and hyperdense mass with homogenous contrast enhancement. The lesion resolved with radiotherapy. What is the most likely diagnosis? [B5 Q42]
(a) Glioma
(b) Metastases
(c) Lymphoma
(d) Sarcoidosis
(e) Oligodendroglioma
Lymphoma
This is seen not only in immunocompromised but also immunocompetent patients. On CT, the lesion is usually hyperdense showing homogenous enhancement. Resolution with steroids and/or radiotherapy is a characteristic finding of cerebral lymphoma.
A 50-year-old male patient is referred from A&E for a CT brain. He has a history of headache for 2 months and increasing clumsiness. The CT shows a lesion abutting the sphenoid in the anterior cranial fossa. It measures 4 cm in size and demonstrates heterogeneous enhancement. There is no evidence of calcification. The lesion is displacing and effacing the third ventricle, causing mild hydrocephalus in the lateral ventricles. An MRI is performed. The lesion is isointense on T1WI and T2WI and demonstrates an enhancing dural tail and broad dural base. Again, the enhancement pattern is heterogeneous. Several flow voids are noted in the lesion. There is little peri-tumoural oedema noted. An MR angiogram is performed, which indicates dual supply to the lesion from the internal and external carotid artery. What lesion typically demonstrates these imaging findings? [B1 Q10]
A. WHO 1 meningioma.
B. WHO 3 meningioma.
C. Haemangiopericytoma.
D. Melanocytoma.
E. Schwannoma
Haemangiopericytoma.
The imaging findings clearly describe an extra-axial tumour. These are classed as either lesions of meningeal origin or tumours of neurogenic origin. The broad dural base and dural tail indicates that this is a tumour of meningeal origin. Meningioma is the most common of these tumours, but the imaging characteristics are not typical. World Health Organization (WHO) 1 and 2 meningiomas demonstrate uniform enhancement, although haemorrhage into meningiomas is recognized. WHO 3 meningiomas (malignant meningiomas) are indicated on imaging when there is invasion of the adjacent parenchyma, which is not described. Otherwise, the diagnosis of malignant meningioma is made by an aggressive pattern of growth on serial imaging and biopsy. The dual arterial supply is also atypical. While meningiomas can have dual supply, they more typically derive their arterial supply from the external carotid (via the meningeal artery). Melanocytomas are suggested on imaging by increased signal on T1WI and are more commonly infratentorial. Without this, the diagnosis is again reached most following biopsy. All the features described are typical of haemangiopericytoma.
A 40-year-old female is investigated for worsening headaches. CT shows a well-defined hyperdense globular lesion within the trigone of the left lateral ventricle. There is intense contrast enhancement. The most likely diagnosis is: [B2 Q2]
a. Choroid cyst
b. Ependymoma
c. Colloid cyst
d. Meningioma
e. Neurocytoma
Meningioma
It is rare for meningiomas to occur intraventricularly (2–5% of all meningiomas) but they are the most common trigonal intraventricular mass in adulthood. They tend to occur in 40-year-old females.
Which of the following favours dural meningeal carcinomatosis rather than leptomeningeal cacinomatosis? [B3 Q21]
A. Short discontinuous thin sections of enhancement
B. Positive cytology
C. Thin area of subarachnoid enhancement following convulsions of gyri
D. Discrete leptomeningeal nodules
E. Invasion of underlying brain with mass effect and oedema
E
Dural meningeal carcinomatosis is rarely associated with positive cytology and involves localised or diffuse curvilinear enhancement underneath inner table in expected position of dura.
Which of the following is the most common radiation-induced CNS tumour? [B4 Q60]
a. ependymoma
b. oligodendroglioma
c. lymphoma
d. glioblastoma multiforme
e. meningioma
Meningioma
Meningioma is the most common radiation-induced CNS tumour and has been particularly associated with low-dose radiation treatment for tinea capitis. For the diagnosis of radiation-induced meningioma to be made, the meningioma must arise in the radiation field, appear after a latency period of years and should not have been the primary tumour irradiated. Radiation-induced meningiomas are more frequently multiple and have higher recurrence rates than non-radiation-induced tumours.
A 50-year-old man presents with headaches. A CT of the head reveals a 3 cm extra-axial lesion in the posterior fossa adjacent to the tentorium. The lesion is iso-dense with brain on non-contrast CT and has small areas of calcifications within. The lesion enhances homogenously with contrast. There is no surrounding oedema in the brain parenchyma. The most likely diagnosis of the abnormality is? [B5 Q3]
(a) Medulloblastoma
(b) Meningioma
(c) Lymphoma
(d) Glioblastoma multiforme
(e) Osteosarcoma metastasis
Meningioma
These are all typical features of a meningioma. Medulloblastoma is a childhood infratentorial tumour which is usually hyperdense and enhances with contrast. Lymphoma can be isodense with homogenous enhancement but usually is associated with significant peritumoral oedema and no calcifications. Glioblastoma multiforme is seen as an irregular lesion with mass effect, oedema and a heterogenous enhancement pattern. Sarcoma metastases are rare and would induce peritumoral oedema.
A 52-year-old man presents with headaches. Head CT shows a 4 cm extra-axial, homogenous, hyperdense lesion which enhances avidly with contrast. There is hyperostosis in the adjacent part of frontal bone. What is the most likely diagnosis? [B5 Q44]
(a) Meningioma
(b) Lymphoma
(c) Metastasis
(d) Glioma
(e) Oligodendroglioma
Meningioma
These tumours arise from the arachnoid ‘cap’ cells of the arachnoid villi.
A 45-year-old woman presents with a several-month history of neck pain and gradually progressive weakness and paraesthesia in the upper limbs. An MRI scan of the cervical spine is performed, and this shows a well-defined central intramedullary mass in the mid-cervical spinal cord. The mass is generally slightly hyperintense on T2WI, but also has a few low signal peripheral areas. It enhances homogeneously with gadolinium. What is the most likely diagnosis? [B1 Q27]
A. Astrocytoma.
B. Metastasis.
C. Haemangioblastoma.
D. Ganglioglioma.
E. Ependymoma
Ependymoma.
Ependymoma is the most common intramedullary neoplasm in adults. It tends to be centrally located within the cord, unlike astrocytoma, which can be eccentric. Astrocytoma can have a longer segment of cord involvement than ependymoma and may have a more infiltrative margin. The peripheral low signal areas seen on T2WI in ependymoma are related to haemosiderin deposition from prior haemorrhage.
A 22-year-old woman presents with upper and lower limb neurological symptoms and signs. She is subsequently discovered on MRI to have a mass in the cervical spinal cord. Which of the following features on MRI is going to point more towards a diagnosis of spinal cord astrocytoma, rather than ependymoma? [B1 Q48]
A. Predominant T2WI high signal.
B. Homogeneous enhancement post gadolinium.
C. Short segment of cord involvement.
D. Eccentrically placed lesion in the cord.
E. Sharply marginated lesion.
D
Eccentrically placed lesion in the cord. Table 6.2 illustrates the diagnostic features of astrocytoma and ependymoma.
A 52-year-old woman presents with gradually increasing gait disturbance and lower limb sensory symptoms. An MRI of her spine is performed, and this shows an anteriorly placed intradural, but extramedullary spinal mass. It is fairly markedly low signal on T1WI and T2WI and shows only minimal patchy enhancement post administration of intravenous gadolinium. What is the most likely diagnosis? [B1 Q49]
A. Neurofibroma.
B. Schwannoma.
C. Lymphoma.
D. Metastasis.
E. Meningioma.
Meningioma.
Spinal meningiomas are typically iso- to hypointense on T1WI and slightly hyperintense on T2WI. There is usually strong and homogeneous enhancement with gadolinium. However, some meningiomas may contain calcification and are typically the only intradural extramedullary tumours to do so. Some meningiomas can be heavily calcified and such a meningioma is being described in the question. These will remain dark on all MRI sequences and demonstrate only little contrast uptake (in the non-calcified areas). Schwannomas, neurofibromas, and metastases would not typically be hypointense on T2WI. Meningeal lymphomas are very rare and usually manifest as diffuse thickening of nerve roots and/or multiple enhancing nodules.
A 35-year-old man attends the accident and emergency department complaining of episodic lower back pain radiating down the legs. History and clinical examination also suggest pelvic sphincter dysfunction. MRI shows a spinal cord mass located at the conus medullaris. The mass is isointense on T1 and hyperintense on T2. It demonstrates contrast enhancement. The most likely diagnosis is: [B2 Q25]
a. Astrocytoma
b. Intradural lipoma
c. Haemangioblastoma
d. Myxopapillary ependymoma
e. Ganglioglioma
Myxopapillary ependymoma
This is a variant of ependymoma and is the most common neoplasm of the conus medullaris. It originates from ependymal glia of the filum terminale. Average age at presentation is 35 years and it is more common in men. T1-weighted imaging shows an isointense or occasionally hyperintense (due to the mucin content) mass. It is hyperintense on T2 and almost always shows enhancement postcontrast. Intradural lipomas are hyperintense on T1-weighted imaging, but they should not enhance. They also tend to occur in younger individuals and usually have an associated, clinically apparent lumbosacral mass. Haemangioblastoma can also demonstrate high signal on T1 but is also highly vascular, can show signal voids and approximately half of them will have an intra-tumoural cystic component. Gangliogliomas are much more common at the cervical and thoracic levels.
A follow-up of a young man under the care of the neurooncologists reveals a drop in metastases. Which statement is most true? [B3 Q28]
A. Drop metastases tend to be in the upper spine
B. Metastases within the spinal canal are usually ventral
C. Glioblastoma is the commonest cause of drop metastases
D. Are associated with positive CSF cytology in approximately 10% of cases
E. Usually demonstrates homogenous enhancement with contrast
E
Drop metastases are usually dorsal in location in the spinal canal. Medulloblastomas are the most common cause of drop metastases. A higher percentage of CSF cytology is positive.
Which is most likely to represent an intramedullary mass lesion? [B3 Q43]
A. Ependymoma
B. Meningioma
C. Neurofibroma
D. Arachnoid cyst
E. Abscess
Ependymoma
Other intramedullary masses include astrocytoma are dermoids (lipoma/teratomas), acutely expanding infarcts and haematoma.
A 50-year-old female presents with increasing weakness of the lower limbs and sensory disturbance. MRI shows an extramedullary, intradural ovoid mass in the mid-thoracic region. The lesion has signal isointense to the cord on T1W and T2W images and enhances avidly with intravenous gadolinium. What is the most likely diagnosis? [B4 Q98]
a. meningioma
b. nerve sheath tumour
c. ependymoma
d. dermoid
e. arachnoid cyst
Meningioma
Meningiomas in the spine are seen mainly in females (80%) and in those over 40 years of age. They have similar signal characteristics to the cord on MRI and enhance avidly. They can cause symptoms related to cord compression. Bone erosion is seen in ,10%. Nerve sheath tumours produce fusiform masses arising from the nerve roots, often extending through the intervertebral foramen, causing them to be dumbbell shaped. They are isointense to muscle on T1W and hyperintense to fat on T2W images. Ependymomas are usually located in the filum terminale and show low/intermediate signal on T1W images, with foci of high signal on T2W images, and they often enhance. Dermoids usually occur in the conus or cauda equina and are associated with spinal dysraphism in one-third of cases. They are variable in signal on T1W but are high signal on T2W images.
A 48-year-old female presents with tinnitus. CT shows a soft-tissue mass in the region of the hypotympanum. There is irregular bone demineralisation in the region of the carotid canal and jugular foramen, making their margins irregular and partially indistinct. On proton density MR imaging, the mass has mixed hyper- and hypo-intensity signal. The tumour shows strong enhancement after gadolinium administration. What is the most likely diagnosis? [B2 59]
a. Glomus tympanicum tumour
b. Glomus jugulare tumour
c. Carotid body tumour
d. Glomus vagale tumour
e. Cholesteatoma
Glomus jugulare tumour
All the tumours listed in the differential (apart from cholesteatoma) are paragangliomas. They grow slowly and rarely metastasise. Glomus jugulare tumours originate from the adventitia of the jugular vein. CT demonstrates a soft-tissue mass in the region of the jugular bulb/hypotympanum/middle ear space. Local bone destruction is common, particularly of the jugular plate or the lateral portion of the carotico-jugular spine. A unique ‘salt-and-pepper’ pattern of hyper- and hypo-intensity on T1- and T2-weighted images is also seen. This represents multiple small tumour vessels. They are highly vascular lesions, usually deriving a blood supply from branches of the external carotid artery.
A 45-year-old man presents with a 4-month history of worsening lower back pain radiating into the right lower extremity with weakness. An MRI scan of lumbar spine shows a 3-cm well-defined ovoid lesion eccentrically placed at the conus medullaris, the location of which is felt to be intradural and extramedullary. It is hypointense on T1WI and hyperintense on T2WI. There are flow voids and hemorrhage within the lesion. It enhances avidly post injection of contrast. Which of the following lesions fits best with the imaging findings? [B1 Q26]
A. Meningioma.
B. Schwannoma.
C. Neurofibroma.
D. Paraganglioma.
E. Metastasis
Paraganglioma.
Paragangliomas are rare intradural extramedullary tumours that are usually benign and have imaging characteristics as those described. Schwannomas, neurofibromas, and meningiomas are much more common, but do not typically contain vascular flow voids or areas of hemorrhage. Meningiomas may have a dural tail or foci of calcification. Schwannomas and neurofibromas can be difficult to distinguish on imaging. They are typically isointense on T1WI and markedly hyperintense on T2WI. Enhancement may be intense and homogenous or peripheral. Neurofibromas tend to encase, rather than displace, nerve roots. Leptomeningeal metastases present with three different imaging patterns: 1) Diffuse, thin enhancing coating of the surface of the spinal cord and nerve roots. 2) Multiple small enhancing nodules on the surface of the cord and nerve roots. 3) A single mass in the lowest part of the thecal sac.
A 60-year-old woman presents with a painless, slowly growing mass in the lateral aspect of the neck. The patient is referred for imaging with a clinical diagnosis of carotid body paraganglioma. Which of the following is a distinctive feature of carotid body paraganglioma on imaging? [B1 Q47]
A. Soft-tissue mass in the carotid space.
B. Intense enhancement after IV contrast administration.
C. High signal on T2WI.
D. Splaying of the internal and external carotid arteries.
E. Low signal on T2WI.
D. Splaying of the internal and external carotid arteries.
Carotid body tumour or paraganglioma is the most common paraganglioma of the head and neck. It arises from the paraganglionic cells located on the medial aspect of the carotid bifurcation. On MRI, they are of low to intermediate signal intensity on T1WI and hyperintense on T2WI. They are hypervascular and demonstrate intense enhancement after contrast administration. Splaying of the internal and external carotid arteries and multiple flow voids producing a ‘salt and pepper’ appearance are distinctive features on imaging.
A 56-year-old male presents with a mass in the anterior triangle of the neck and is referred for ultrasound scan and biopsy. Ultrasound scan shows a vascular mass splaying the internal and external carotid arteries at their origin. How should cell sampling of the mass be undertaken? [B4 Q64]
a. performs FNA rather than core biopsy
b. uses multiple passes
c. uses a single pass
d. ensures that needle midway between internal and external carotid arteries
e. both FNA and core biopsy contraindicated
Both FNA and core biopsy contraindicated
The imaging appearances are highly suggestive of a carotid body tumour (paraganglioma), and biopsy should not be performed, as hemorrhage would be certain due to the high vascularity of the lesion (in addition to the proximity of the carotid system). They present as painless, pulsatile masses in the neck of adults, below the angle of the jaw, and are laterally mobile but vertically fixed. There is splaying of the carotid bifurcation but preservation of calibre of the two arteries. Contrast imaging shows avid enhancement.
A 40-year-old hypertensive woman presents with a neck mass. A left carotid angiogram demonstrates an intensely enhancing mass splaying the carotid bifurcation. What is the most likely diagnosis? [B5 Q30]
(a) Metastasis
(b) Carotid body paraganglioma
(c) Lymphoma
(d) Branchial cyst
(e) Carotid dissection
Carotid body paraganglioma
Carotid body tumours are the most common extracranial head and neck paragangliomas. These typically splay the internal and external carotid arteries because they arise from the tissue located at the carotid artery bifurcation. They demonstrate an intense and persistent vascular blush on imaging. The combination of intense blush with flow voids on MRI has been described as a ‘salt and pepper’ appearance. These tumours may be familial and multicentric and are malignant in 10% cases.
A 45-year-old female patient is referred to you with a history of mild confusion and new onset seizures. This patient has a history of breast cancer and is currently undergoing chemotherapy following a wide local excision with axillary radiotherapy. The patient has a low-grade pyrexia and a low neutrophil count. An MRI reveals a solitary lesion in the right frontal lobe, which is low/intermediate signal on T1WI and high signal on T2WI. The lesion demonstrates ring enhancement. Diffusion-weighted imaging (DWI) reveals slightly increased signal on B1000 imaging, with increased signal on B0 imaging, and this area is bright on the apparent diffusion coefficient (ADC) map. What is the most likely diagnosis? [B1 Q3]
A. Abscess.
B. Metastasis.
C. Glioblastoma multiforme.
D. Infarct.
E. Radiotherapy change.
Metastasis.
The answer options give some of the classical radiological differentials for a ring-enhancing lesion in the brain: MAGIC DR (metastasis, abscess, glioblastoma multiforme, infarct, contusion, demyelinating conditions, and post-radiotherapy change). The clinical history should steer the reader toward the first two. DWI is reasonably useful at differentiating between these two (exceptions being in some fungal infections and toxoplasmosis), with necrotic metastasis having unrestricted diffusion and abscess having restricted diffusion. The appearances in this case are slightly confused by the DWI b1000 imaging showing increased signal. However, the presence of increased signal on the ADC map correctly identifies this as being T2 shine through, showing the importance of checking both sequences.
A 52-year-old man presents following collapse. He was previously fit and well, describing only a relatively recent history of dull back pain. Initial CT scan of the head reveals a 1.5 cm hyperdense mass at the corticomedullary junction of the right cerebral hemisphere. The mass shows surrounding oedema which exceeds the volume of the lesion. There is strong lesional enhancement following contrast injection. What is the most likely diagnosis and subsequent management? [B2 Q21]
a. Glioblastoma multiforme with subsequent MRI of the brain
b. Prostatic cancer metastasis with digital rectal examination and measuring of the prostatic specific antigen
c. Acute haemorrhagic contusion with referral to the neurosurgeons for active monitoring
d. Renal cell metastasis with subsequent CT staging
e. Brain abscess with subsequent intravenous antibiotics
Renal cell metastasis with subsequent CT staging
Brain metastases account for approximately a third of all intracranial tumours and are the most common intracranial neoplasm. They characteristically occur at the corticomedullary junction of the brain and have surrounding oedema that typically exceeds the tumour volume. Multiple lesions are present in approximately two-thirds of cases and should be searched for with administration of intravenous contrast. Most are hypodense on CT unless haemorrhagic or hypercellular, hence the lesion in this case is haemorrhagic. This lends itself to a differential of primary neoplasms which includes melanoma, renal cell carcinoma, thyroid carcinoma, bronchogenic carcinoma, and breast carcinoma. The history of back pain also suggests bone metastases. Glioblastoma multiforme usually appears as an irregular, heterogeneous, low-density mass. Abscesses typically demonstrate ring enhancement post-contrast and may show loculation and specules of gas. The patient’s history describes collapse rather than headache or confusion following a fall, which moves the differential away from traumatic contusion. Although prostate cancer typically metastasises to the vertebrae, it is an uncommon primary site for brain metastases, especially as the lesion described is haemorrhagic.
A 65-year-old woman is investigated for enophthalmos and headache. She is cachetic, anaemic and you suspect a metastatic process. CT head demonstrates an infiltrative retrobulbar mass. What is the most likely site of primary disease? [B2 Q33] {will repeat again in orbit session}
a. Breast
b. Lung
c. Renal
d. Melanoma
e. Ovarian
Breast
Most retrobulbar metastases are extraconal (outside the muscle cone). Neuroblastoma and Ewing’s sarcoma are the most common in children and produce smooth extraconal masses related to the posterior lateral wall of the orbit. In adults, an infiltrative retrobulbar mass and enophthalmos is characteristic of scirrhous carcinoma of the breast (invasive ductal carcinoma). Enophthalmia is also considered to be one of the earliest signs of metastatic breast cancer.
Which of the following favours dural meningeal carcinomatosis rather than leptomeningeal cacinomatosis? [B3 Q21]
A. Short discontinuous thin sections of enhancement
B. Positive cytology
C. Thin area of subarachnoid enhancement following convulsions of gyri
D. Discrete leptomeningeal nodules
E. Invasion of underlying brain with mass effect and oedema
Short discontinuous thin sections of enhancement
Dural meningeal carcinomatosis is rarely associated with positive cytology and involves localised or diffuse curvilinear enhancement underneath inner table in expected position of dura.
Which is the cause of a cystic rather than a haemorrhagic cause of brain metastases? [B3 Q22]
A. Adenocarcinoma of the lung
B. Malignant melanoma
C. Choriocarcinoma
D. Renal cell carcinoma
E. Thyroid carcinoma
Adenocarcinoma of the lung
Squamous cell lung cancer and adenocarcinoma of the lung cause cystic metastasis to the brain. Answers B-E are causes of haemorrhagic metastases.
A 64-year-old woman presents with progressive headache and confusion. On CT, she is found to have multiple, well-defined, rounded, low-attenuation masses of varying sizes in both hemispheres at the grey–white matter junction. The masses demonstrate intense enhancement following intravenous contrast, and there is considerable surrounding oedema. Which of the following is the most appropriate next imaging investigation? [B4 Q19]
a. mammography
b. thyroid ultrasound scan
c. barium enema
d. renal ultrasound scan
e. chest radiograph
Chest Radiograph
Brain metastases are the most common intracranial tumours. Six primary tumours account for 95% of all brain metastases. Primary bronchial carcinoma is the most common (47% of cases), though squamous cell carcinoma rarely metastasizes to the brain. Other common primary tumours are breast carcinoma (17%), gastrointestinal malignancy (15%), renal cell carcinoma, melanoma and choriocarcinoma. Metastases characteristically occur at the grey–white matter junction, are multiple in 66% of cases, and typically appear as hypodense masses that demonstrate solid or ring enhancement.
A 42-year-old Caucasian woman presents with multiple fits. CT of the head shows multiple, small enhancing lesions in the cortical and subcortical areas. On MRI, these lesions return low signal on T2 and hyperintense on post-gadolinium T1. What is the most likely diagnosis? [B5 Q43]
(a) Tuberous sclerosis
(b) Calcifications
(c) Melanoma metastases
(d) Haemorrhagic metastases
(e) Lymphoma
Melanoma metastases
The T2 shortening effect is attributed to the paramagnetic effects of iron and copper bound to melanin.
A 30-year-old male patient attends A&E 30 minutes after a head injury. He has consumed alcohol. You are contacted by the A&E doctor, who requests a CT brain. Currently, which of the following is a correct indication for immediate scanning? [B1 Q28]
A. Two episodes of vomiting.
B. GCS 13.
C. Loss of consciousness.
D. Amnesia for 20 minutes before accident.
E. Visual hallucinations.
Two episodes of vomiting.
Almost anything will buy you a CT brain these days, but surprisingly, the GCS must be less than 13 in the A&E department within 2 hours of the injury under NICE guidelines to warrant one. After 2 hours have passed, a GCS of anything less than normal is an indication for scanning. Other factors which require a CT within 1 hour are suspected open or depressed skull fracture, sign of fracture at skull base, post-traumatic seizure, focal neurological deficit and amnesia, or loss of consciousness and coagulopathy. Additional factors which necessitate a CT brain within 8 hours are amnesia of events for greater than 30 minutes before impact, and if there is any amnesia or loss of consciousness and the patient is older than 65/dangerous mechanism of injury. Note that, perversely, even if the scan can be delayed but be performed within 8 hours, the guidelines state it should be requested immediately. Prepare to be awoken from your sleep!
A 27-year-old female patient undergoes urgent neuroimaging following loss of consciousness because of an RTA. CT is unremarkable. MRI reveals multiple small areas of increased signal on T2WI in the white matter near the grey–white matter junction within the frontal and temporal lobes. In the same locations, DWI reveals areas of increased signal on the B1000 image and reduced signal on the ADC map. What is the most likely diagnosis? [B1 Q29]
A. Subarachnoid haemorrhage.
B. Extradural haematoma.
C. Subdural haematoma.
D. Hypoxic brain injury.
E. Diffuse axonal injury
Diffuse axonal injury (DAI).
CT is initially often normal (up to 80% of cases) in DAI. If positive, it may reveal small low attenuation foci (oedema) or high attenuation foci of petechial haemorrhage. The gray/white matter interface of the frontotemporal lobes, corpus callosum (especially the splenium), and brainstem are the most involved sites in DAI. MRI is much more sensitive and is the investigation of choice. The signal on MRI depends on the age of the lesion and whether haemorrhage is present, but classically hyperintense foci on T2WI sequences are seen acutely. In the more chronic phase, the lesions may only be detected as hypointense foci at characteristic locations on GE sequences: this appearance may remain for years. DWI reveals hyperintense foci of restricted diffusion on B1000 images, with corresponding low signal on the ADC map. The findings on DWI are easily distinguishable from extradural haematoma/subarachnoid haemorrhage/subdural haematoma/generalized oedema, which are discussed in other questions in this chapter.
A 44-year-old woman presents with severe facial injuries following an RTA. CT of the facial bones demonstrates multiple maxillofacial fractures in keeping with a Le Fort configuration. Which of the following statements regarding Le Fort fractures is false? [B1 Q51]
A. Any combination of Le Fort I, II, and III fractures can occur.
B. Disruption of pterygoid plates from the posterior maxilla is an essential finding in Le Fort fractures.
C. Le Fort fractures, by definition, refer to fractures involving the maxilla bilaterally.
D. Craniofacial separation is noted in Le Fort III pattern.
E. Le Fort fracture associated with a palate fracture will result in widening of the maxillary arch.
Le Fort fractures by definition refer to fractures involving the maxilla bilaterally.
Separation of all or a portion of the maxilla from the skull base is described as a Le Fort fracture. This can be unilateral when it is associated with sagittal or parasagittal fractures of the palate. Le Fort fractures by definition involve the posterior maxillary buttress at the junction of the posterior maxillary sinus and the pterygoid plates of the sphenoid. This may be either through the pterygoid plates or through the posterior walls of the maxillary sinus. Once a pterygomaxillary disruption has been identified, the remaining facial buttresses are inspected to identify the type of Le Fort fracture. In Le Fort I fracture, the maxillary arch will move in relation to the rest of the face and skull. In Le Fort II fracture, the entire maxilla will move in relation to the skull base. In Le Fort III, there is complete craniofacial separation. Any combination of Le Fort I, II, and III can occur. Posterior extension of Le Fort fracture into the hard palate results in widening of the maxillary arch and dental malocclusion.
A 40-year-old man with no previous medical history or medication attends the accident and emergency department. He was the driver of a car that was involved in a car-on-car vehicle collision at approximately 40mph. He was wearing a seat belt and his airbag deployed appropriately. According to NICE guidelines for head injury, which one of the following criteria alone does not warrant an acute head CT scan? [B2 Q34]
a. GCS <15 when he was assessed in the emergency department two hours after the accident
b. Haemotympanum
c. Amnesia of events <30 minutes after impact
d. Seizure following the accident
e. More than one episode of vomiting following the accident
Amnesia of events <30 minutes after impact
NICE defines head injury as ‘any trauma to the head, other than superficial injuries to the face’. All of the other criteria listed are requisites for an acute head CT scan. Haemotympanum implies a basal skull fracture which should be investigated by CT. Amnesia of events >30 minutes before impact would require an acute head CT. Any amnesia or loss of consciousness since the injury requires a CT scan if the patient is equal to or older than 65 years, has a coagulopathy (including warfarin treatment) or if there is a history of dangerous mechanism of injury, which is listed as:
* Pedestrian or cyclist struck by a motor vehicle.
* Occupant ejected from a motor vehicle.
* Fall from over one metre or five stairs.
A 24-year-old man is an unrestrained passenger in a car involved in a high-speed collision. He is found unconscious at the scene. CT of the brain is normal. MR scan of the brain demonstrates multiple small foci of high signal on T2W images in the white matter of the parasagittal regions of the frontal lobes and the periventricular regions of the temporal lobes. What is the most likely diagnosis? [B4 Q80]
a. acute subdural haematoma
b. diffuse axonal injury
c. cortical contusions
d. intracerebral haematoma
e. subcortical grey matter injury
b. diffuse axonal injury
Diffuse axonal injury is characterized by widespread axonal disruption occurring in response to acceleration or deceleration forces – direct impact is not necessary. Typically, patients are immediately unconscious after the injury. CT is commonly negative, though 20% of lesions contain sufficient haemorrhage to be visible. On MR scan, typical findings are of multiple small foci of decreased signal intensity on T1W images and increased signal intensity on T2W images. Characteristic locations are the frontal and temporal white matter near the grey–white matter junction. More severe injuries may involve the lobar white matter and corpus callosum, with the brain stem involved in the most severe cases. Cortical contusions usually involve the superficial grey matter, and patients are less likely to present with immediate loss of consciousness. They characteristically occur near bony protuberances and are more commonly haemorrhagic. Subcortical grey matter injury is an uncommon type of injury seen after severe head trauma, with petechial haemorrhages in the basal ganglia and thalamus.
A 32-year-old man presents with recent onset of migraine and TIAs. He also reports some cognitive decline. Cerebral angiogram is normal. An MRI of brain reveals discrete hyperintensities in the anterior temporal poles and external capsules. What is the most likely diagnosis? [B1 Q8]
A. Cerebral autosomal dominant arteriopathy with subcortical infarcts and leukoencephalopathy (CADASIL).
B. Mitochondrial myopathy, encephalopathy, lactic acidosis, and stroke (MELAS).
C. Myoclonic epilepsy with ragged red fibres (MERRF).
D. Sporadic subcortical arteriosclerotic encephalopathy (sSAE).
E. Protein S deficiency
A. Cerebral autosomal dominant arteriopathy with subcortical infarcts and leukoencephalopathy (CADASIL).
This is a hereditary small vessel disease, which causes stroke in young adults. The genetic mutation is found on chromosome 19. Presentation can include migraine, cognitive decline, psychiatric disturbance, TIAs, and stroke, the latter usually with substantial/complete recovery after individual strokes, particularly early in the disease process. Imaging reveals subcortical lacunar infarcts and leukoencephalopathy in young adults. The frontal lobe has the highest lesion load, followed by the temporal lobe and insula. Anterior temporal pole and external capsule lesions have higher sensitivity and specificity for CADASIL. The cerebral cortex is usually spared. MRI is the investigation of choice: CT will reveal only areas of hypodensity, and angiography is normal. sSAE is associated with hypertension and results in multiple lacunar infarcts in the lenticular nuclei, pons, internal capsule, and caudate nuclei. MELAS and MERRF are mitochondrial disorders. MELAS results in bilateral multiple cortical and subcortical hyperintense lesions on FLAIR images; MERRF has a propensity for the basal ganglia and caudate nuclei, and watershed ischaemia/infarcts are common. Hypercoagulable states such as protein S deficiency result in cortical and lacunar infarcts of various sizes, but the cerebral angiogram is abnormal.
A 74-year-old woman is admitted with acute left-sided hemiplegia of 2 hours onset. On assessment by the stroke team, she is deemed suitable for thrombolysis and referred for CT. As part of your institution’s work-up, CT perfusion (CTP) is performed following the unenhanced study. Which of the following CTP findings with regards to cerebral blood flow (CBF), cerebral blood volume (CBV) and mean transit time (MTT) are consistent with infarction? [B1 Q59]
A. Decreased CBF/decreased CBV/increased MTT.
B. Decreased CBF/increased CBV/increased MTT.
C. Decreased CBF/decreased CBV/decreased MTT.
D. Increased CBF/increased CBV/decreased MTT.
E. Increased CBF/decreased CBV/decreased MTT.
A. Decreased CBF/decreased CBV/increased MTT.
In identifying the ischaemic penumbra, CT perfusion offers promise in improved patient selection for thrombolysis beyond a rigid time window. In the infarct core (tissue which is not salvageable) both CBF and CBV are decreased with a corresponding increase in MTT. Penumbral tissue (which is potentially recoverable by thrombolysis) exhibits a CBF/CBV mismatch with an increased CBV, but decreased CBF (and increased MTT). An increase in CBF and CBV with decreased MTT is a feature noticed in tumours secondary to angiogenesis and microvascular permeability.
A 74-year-old man with a history of diabetes presents with a history of slurred speech and right-sided weakness, which spontaneously resolved over 1–2 hours. Initial unenhanced CT is normal. Further investigation by MRI is requested. What sequence is most sensitive for diagnosing acute ischaemia? [B1 Q66]
A. T2WI.
B. FLAIR.
C. DWI.
D. GE imaging.
E. T1WI post gadolinium
C. DWI.
The sensitivity and specificity of DWI in detecting acute ischaemia are 88–100% and 86–100%, respectively, and DWI is established as better than other conventional MRI techniques. In addition, acute from chronic ischaemia can be differentiated. DWI is a measure of the Brownian motion of water molecules within a tissue. Two sets of images are acquired: the DWI and the quantitative ADC map. Chronic lesions may appear bright on the DWI (T2 shine through), but only acute lesions with restricted diffusion will be dark on the ADC map. It should be noted, however, that high signal intensity on DWI with low ADC has been reported in a variety of conditions such as abscess, lymphoma, and gliomas. GE is useful for identifying microbleeds that may influence antithrombotic treatment.
You are called by a paediatrician to perform a cranial ultrasound on a term neonate who requires intensive therapy following delivery. Ultrasound demonstrates a well-defined area of increased parenchymal echogenicity over the periphery of the right parietal lobe. What is the most likely diagnosis? [B2 Q14]
a. Germinal matrix haemorrhage
b. Venous infarction
c. Middle cerebral artery infarction
d. Periventricular leukomalacia
e. Subarachnoid haemorrhage
c. Middle cerebral artery infarction
Vascular occlusive disease is rare in the neonatal period. When present, it is more common in the term infant and usually results from thrombosis rather than embolism. The MCA is most involved. Aetiology includes traumatic delivery, vasospasm due to meningitis and emboli secondary to congenital heart disease. Ultrasound demonstrates echogenic parenchyma in the distribution of the arterial territory.
Which of the following best describes the appearance of an acute ischaemic infarct of the brain? [B2 Q22]
a. Hypointense on DW MRI and low apparent diffusion coefficient (ADC) values.
b. Hyperintense on DW MRI and low ADC values
c. Hypointense on DW MRI and high ADC values
d. Hyperintense on DW MRI and high ADC values
e. Isointense on DW MRI and low ADC values
b. Hyperintense on DW MRI and low ADC values
Diffusion-weighted imaging is dependent on the motion of water molecules and provides information on tissue integrity. It is thought that interruption of cerebral blood flow results in rapid breakdown of energy metabolism and ion exchange pumps. This causes a shift of water from the extracellular compartment into the intracellular compartment, giving cytotoxic odema. This produces the hyperintensity on DW MR images. ADC values tend to be low within hours of stroke and continue to decline for the next few days. They remain reduced through the first four days and then show pseudonormalisation between four and ten days. After ten days the ADC tends to rise. Hyperintensity on diffusion weighted (DW) MRI and low ADC values are not pathognomonic of acute infar
Considering Moya-Moya disease in adults: [B3 Q41]
A. Infarct seen in cortical/subcortical areas
B. Multiple small flow voids are characteristic
C. Affects anterior circulation
D. Presentation with ischemia is more common in adults than in children
E. The supra-clinoid MCA is spared
B
Multiple flow voids are due to hypertrophied lenticulostriate arteries. Children more often present with ischemia and infarct in cortical/subcortical areas. Adults more often present with haemorrhage than children, but when infarcts do occur, they are most often in the deep white matter. The disease can involve the posterior circulation. The supra-clinoid MCA is the first to be involved.
Carotid Doppler examination is performed on a patient following a transient ischaemic attack. All other factors are in favour of surgical treatment for carotid stenosis. At what peak internal carotid artery velocity would ultrasound scan also support this management? [B4 Q68]
a. 50 cm/s
b. 75 cm/s
c. 100 cm/s
d. 200 cm/s
e. 250 cm/s
250 cm/s
Carotid ultrasound scan can be used to assess the common, external, and internal carotid arteries and the carotid bulb, including the vessel walls and the presence of plaques and stenoses. Doppler scan can display velocity profiles and allow waveform analysis and peak velocity measurement. Flow velocity increases proportionally with the degree of stenosis, except when the affected vessel is almost completely or totally occluded. Here flow velocity drops off. Stenosis above 70% is considered for surgery. This corresponds to a flow rate of greater than 250 cm/s.
A 30-year-old male with recurrent Transient Ischemic Attacks (TIAs) and a history of migraine with aura undergoes CT brain. Subcortical infarcts are identified raising suspicion of cerebral autosomal dominant arteriopathy with subcortical infarcts (CADASIL). Which is the most characteristically involved location for subcortical infarcts? [B3 Q3]
A. Anterior temporal pole
B. Frontal lobe
C. Centrum semi-ovale
D. Deep grey matter structure
E. Pons
Anterior temporal pole
A young patient with migraines, auras, TIAs or subcortical strokes should raise suspicion of CADASIL. Subcortical infarcts are characteristically in the anterior temporal pole and external capsule but may involve C, D and E.
An 80-year-old man presents acutely with a dense hemiplegia. CT perfusion is performed soon after admission, which suggests that the entire involved arterial territory is beyond recovery. Which of the following options represents the most likely combination of cerebral blood flow, mean transit time and cerebral blood volume, respectively, seen within the affected brain parenchyma, compared with unaffected parenchyma? [B4 Q4]
a. increased, increased, increased
b. increased, increased, decreased
c. increased, decreased, decreased
d. decreased, decreased, decreased
e. decreased, increased, decreased
Decreased, Increased, Decreased
Cerebral perfusion CT can distinguish viable but ischaemic tissue (the penumbra) from tissue that is beyond recovery. Other uses include evaluation of vasospasm after subarachnoid haemorrhage, assessment of cerebrovascular reserve with acetazolamide (cerebral arteriole vasodilator) in cases of vascular stenosis, evaluation of collateral flow and cerebrovascular reserve in patients having temporary balloon occlusion and assessment of microvascular permeability of intracranial neoplasms. Cerebral perfusion CT utilizes the central volume principle. This states that CBF=CBV/MTT, where CBF is cerebral blood flow, CBV is cerebral blood volume and MTT is mean transit time. In practice, two CT perfusion techniques can be used. One is perfused-blood volume mapping, in which a quantity is assigned to cerebral blood volume by subtracting unenhanced CT data from CT angiographic data. It has the advantage of imaging the whole brain. The second technique is a dynamic, contrast-enhanced technique that acquires data from a limited number of axial slices and monitors the first pass of an iodinated contrast agent bolus through the cerebral circulation. This requires an unenhanced CT brain, followed by a dynamic CT performed during injection of 50 ml of iodinated contrast (300 mg I/ml) at 4 ml/s. The first pass of contrast is observed in the brain. Cerebral perfusion is related to the concentration of iodinated contrast, which is directly related to the attenuation measured. Several maps are produced, including the CBV, CBF and MTT. MTT is derived from arterial and venous enhancement curves, measured by using regions of interest placed on an artery (one that is not occluded as part of an acute event) and a venous sinus. CBV is the area under the enhancement curves, and CBF is obtained from the central volume equation. Differentiation of infarcted brain from penumbra is important because, while penumbra can be saved by timely thrombolysis, infarcted tissue has an increased risk of bleeding from thrombolysis with no chance of recovery. CBF is decreased in both ischaemia and infarction, MTT is longer (.6 s) in both, while CBV is decreased in infarct but increased (or normal) in the penumbra due to cerebral autoregulatory mechanisms. MTT is the most sensitive for stroke. So, this or CBF can be used to detect stroke while CBV is used to determine whether there is infarct or reversible ischaemia.
A 75-year-old man undergoes an aortic valve replacement. His GCS remains low as the general anaesthetic effect wears off, and a new left-sided weakness is observed. Unenhanced CT of the brain is performed. The CT reveals a hypodensity (of attenuation value–30 HU) within the first segment of the right middle cerebral artery. Which of the following options is most likely to explain the patient’s abnormal neurology? [B4 Q9]
a. right middle cerebral artery thromboembolic occlusion
b. cerebral embolism as part of the fat embolism syndrome
c. right middle cerebral artery fat embolism
d. right middle cerebral artery air embolism
e. right middle cerebral artery dissection
Right Middle Cerebral Artery Fat Embolism
The hypodense artery sign is described, representing a single, large, macroscopic fat embolus within the middle cerebral artery, giving rise to a stroke syndrome. It may occur during cardiac surgery, resulting in dislodgement of fat from the surrounding tissue. This is distinct from the shower of microscopic fat emboli that occurs in the fat embolism syndrome.
A 70-year-old man is referred for CT scan of the brain due to sudden onset of left-sided hemiparesis and clinical diagnosis of stroke. Unenhanced CT shows a rounded area of low attenuation in the right posterior frontal lobe with local gyriform swelling and sulcal effacement. Upon questioning, he reveals a history of lung resection for malignancy 18 months previously. Which of the following imaging investigations would you perform next? [B4 Q25]
a. no further imaging
b. CT of the thorax
c. CT of the thorax and abdomen
d. MRI of the brain
Contrast-Enhanced CT of the brain
In this scenario, the low attenuation and surrounding changes most likely represent brain oedema. This may be due to an evolving infarction or oedema around a metastatic deposit from the previous lung cancer. Differentiation between the two will immediately affect patient treatment, as anti-platelet therapy for ischaemic stroke will increase the risk of haemorrhage from a metastasis and therefore should be withheld if a metastatic deposit is suspected or diagnosed. The primary factor in determining whether a lesion will enhance on CT after administration of intravenous iodinated contrast is the integrity of the blood–brain barrier in that region of the brain substance. A large molecule such as iodinated contrast would not be able to enter the brain unless the integrity of the barrier were compromised. Many aggressive tumours, including metastases, will disrupt this barrier, and so contrast enhancement will be seen in the solid component of these lesions. Acute infarction will typically not show areas of enhancement.
Considering Moya-Moya disease in adults: [B3 Q41]
A. Infarct seen in cortical/subcortical areas
B. Multiple small flow voids are characteristic
C. Affects anterior circulation
D. Presentation with ischemia is more common in adults than in children
E. The supra-clinoid MCA is spared
B
Multiple flow voids are due to hypertrophied lenticulostriate arteries. Children more often present with ischemia and infarct in cortical/subcortical areas. Adults more often present with haemorrhage than children, but when infarcts do occur, they are most often in the deep white matter. The disease can involve the posterior circulation. The supra-clinoid MCA is the first to be involved.
Carotid Doppler examination is performed on a patient following a transient ischaemic attack. All other factors are in favour of surgical treatment for carotid stenosis. At what peak internal carotid artery velocity would ultrasound scan also support this management? [B4 Q68]
a. 50 cm/s
b. 75 cm/s
c. 100 cm/s
d. 200 cm/s
e. 250 cm/s
e. 250 cm/s
Carotid ultrasound scan can be used to assess the common, external, and internal carotid arteries and the carotid bulb, including the vessel walls and the presence of plaques and stenoses. Doppler scan can display velocity profiles and allow waveform analysis and peak velocity measurement. Flow velocity increases proportionally with the degree of stenosis, except when the affected vessel is almost completely or totally occluded. Here flow velocity drops off. Stenosis above 70% is considered for surgery. This corresponds to a flow rate of greater than 250 cm/s.
A 30-year-old male with recurrent Transient Ischemic Attacks (TIAs) and a history of migraine with aura undergoes CT brain. Subcortical infarcts are identified raising suspicion of cerebral autosomal dominant arteriopathy with subcortical infarcts (CADASIL). Which is the most characteristically involved location for subcortical infarcts? [B3 Q3]
A. Anterior temporal pole
B. Frontal lobe
C. Centrum semi-ovale
D. Deep grey matter structure
E. Pons
A. Anterior temporal pole
A young patient with migraines, auras, TIAs or subcortical strokes should raise suspicion of CADASIL. Subcortical infarcts are characteristically in the anterior temporal pole and external capsule but may involve C, D and E.
An 80-year-old man presents acutely with a dense hemiplegia. CT perfusion is performed soon after admission, which suggests that the entire involved arterial territory is beyond recovery. Which of the following options represents the most likely combination of cerebral blood flow, mean transit time and cerebral blood volume, respectively, seen within the affected brain parenchyma, compared with unaffected parenchyma? [B4 Q4]
a. increased, increased, increased
b. increased, increased, decreased
c. increased, decreased, decreased
d. decreased, decreased, decreased
e. decreased, increased, decreased
d. decreased, decreased, decreased
Cerebral perfusion CT can distinguish viable but ischaemic tissue (the penumbra) from tissue that is beyond recovery. Other uses include evaluation of vasospasm after subarachnoid haemorrhage, assessment of cerebrovascular reserve with acetazolamide (cerebral arteriole vasodilator) in cases of vascular stenosis, evaluation of collateral flow and cerebrovascular reserve in patients having temporary balloon occlusion and assessment of microvascular permeability of intracranial neoplasms. Cerebral perfusion CT utilizes the central volume principle. This states that CBF=CBV/MTT, where CBF is cerebral blood flow, CBV is cerebral blood volume and MTT is mean transit time. In practice, two CT perfusion techniques can be used. One is perfused-blood volume mapping, in which a quantity is assigned to cerebral blood volume by subtracting unenhanced CT data from CT angiographic data. It has the advantage of imaging the whole brain. The second technique is a dynamic, contrast-enhanced technique that acquires data from a limited number of axial slices and monitors the first pass of an iodinated contrast agent bolus through the cerebral circulation. This requires an unenhanced CT brain, followed by a dynamic CT performed during injection of 50 ml of iodinated contrast (300 mg I/ml) at 4 ml/s. The first pass of contrast is observed in the brain. Cerebral perfusion is related to the concentration of iodinated contrast, which is directly related to the attenuation measured. Several maps are produced, including the CBV, CBF and MTT. MTT is derived from arterial and venous enhancement curves, measured by using regions of interest placed on an artery (one that is not occluded as part of an acute event) and a venous sinus. CBV is the area under the enhancement curves, and CBF is obtained from the central volume equation. Differentiation of infarcted brain from penumbra is important because, while penumbra can be saved by timely thrombolysis, infarcted tissue has an increased risk of bleeding from thrombolysis with no chance of recovery. CBF is decreased in both ischaemia and infarction, MTT is longer (.6 s) in both, while CBV is decreased in infarct but increased (or normal) in the penumbra due to cerebral autoregulatory mechanisms. MTT is the most sensitive for stroke. So, this or CBF can be used to detect stroke while CBV is used to determine whether there is infarct or reversible ischaemia.
A 75-year-old man undergoes an aortic valve replacement. His GCS remains low as the general anaesthetic effect wears off, and a new left-sided weakness is observed. Unenhanced CT of the brain is performed. The CT reveals a hypodensity (of attenuation value–30 HU) within the first segment of the right middle cerebral artery. Which of the following options is most likely to explain the patient’s abnormal neurology? [B4 Q9]
a. right middle cerebral artery thromboembolic occlusion
b. cerebral embolism as part of the fat embolism syndrome
c. right middle cerebral artery fat embolism
d. right middle cerebral artery air embolism
e. right middle cerebral artery dissection
c. right middle cerebral artery fat embolism
The hypodense artery sign is described, representing a single, large, macroscopic fat embolus within the middle cerebral artery, giving rise to a stroke syndrome. It may occur during cardiac surgery, resulting in dislodgement of fat from the surrounding tissue. This is distinct from the shower of microscopic fat emboli that occurs in the fat embolism syndrome.
A 70-year-old man is referred for CT scan of the brain due to sudden onset of left-sided hemiparesis and clinical diagnosis of stroke. Unenhanced CT shows a rounded area of low attenuation in the right posterior frontal lobe with local gyriform swelling and sulcal effacement. Upon questioning, he reveals a history of lung resection for malignancy 18 months previously. Which of the following imaging investigations would you perform next? [B4 Q25]
a. no further imaging
b. CT of the thorax
c. CT of the thorax and abdomen
d. MRI of the brain
e. contrast enhanced CT of the brain
e. Contrast-Enhanced CT of the brain
In this scenario, the low attenuation and surrounding changes most likely represent brain oedema. This may be due to an evolving infarction or oedema around a metastatic deposit from the previous lung cancer. Differentiation between the two will immediately affect patient treatment, as anti-platelet therapy for ischaemic stroke will increase the risk of haemorrhage from a metastasis and therefore should be withheld if a metastatic deposit is suspected or diagnosed. The primary factor in determining whether a lesion will enhance on CT after administration of intravenous iodinated contrast is the integrity of the blood–brain barrier in that region of the brain substance. A large molecule such as iodinated contrast would not be able to enter the brain unless the integrity of the barrier were compromised. Many aggressive tumours, including metastases, will disrupt this barrier, and so contrast enhancement will be seen in the solid component of these lesions. Acute infarction will typically not show areas of enhancement.
An unenhanced CT scan of the brain is performed 3 hours after the onset of signs suggestive of ischaemic stroke. Which of the following image window parameters is most likely to reveal the early CT changes? [B4 Q34]
a. width 400 HU, centre 40 HU
b. width 80 HU, centre 20 HU
c. width 8 HU, centre 32 HU
d. width 0 HU, centre 0 HU
e. width 1500, centre–500 HU
c. Width 8 HU, centre 32 U
A 75-year-old woman presents with sudden onset left homonymous superior quadrantanopia. Head CT shows a subtle hypo-intensity in the right medial occipital lobe. On MRI, the occipital region shows high signal on FLAIR. Diffusion weighted images show high signal in the right medial occipital lobe consistent with reduced diffusion. What is the most likely diagnosis? [B5 Q32]
(a) Acute occipital lobe infarct in posterior cerebral artery territory
(b) Occipital lobe tumour
(c) Haemorrhage
(d) Old occipital lobe infarct
(e) Acute occipital lobe infarct in middle cerebral artery territory
Acute occipital lobe infarct in posterior cerebral artery territory
Homonymous visual field defects result from retrochiasmal lesions, which include the optic tracts, lateral geniculate body, optic radiations, and visual cortex. Unilateral posterior cerebral artery infarction results in isolated hemianopia. The superior portion of the visual field projects to the inferior occipital cortex. Thus, a partial or branch occlusion of the posterior cerebral artery may lead to an isolated superior or inferior visual field defect.
A 69-year-old lady was admitted 10 days ago following an acute intracerebral haematoma diagnosed on CT. What are the most likely radiological findings on the follow-up MRI scan of brain? [B1 Q4]
A. Haematoma hypointense to grey matter on T1WI, hyperintense on T2WI.
B. Haematoma hyperintense to grey matter on both T1WI and T2WI.
C. Haematoma hyperintense to grey matter on T1WI, hypointense on T2WI.
D. Haematoma hypointense to grey matter on both T1WI and T2WI.
E. Haematoma isointense to grey matter on both T1WI and T2WI.
Haematoma hyperintense to grey matter on both T1WI and T2WI.
The MRI appearances of intracranial haemorrhage are determined primarily by the state of the haemoglobin (Hb), which evolves with age. This can be staged as hyperacute (first few hours), acute (1–3 days), early subacute (3–7 days), late subacute (4–7 days to 1 month), or chronic (1 month to years).
A 68-year-old male attends the emergency department after being found slumped in his chair at home by a carer. CT head shows an intraparenchymal bleed. On MRI the lesion is hyperintense on T1 and hypointense on T2-weighted imaging. Which of the following stages of haemorrhage best correlates with the MRI findings? [B2 Q10]
a. Oxyhaemoglobin
b. Deoxyhaemoglobin
c. Intracellular methaemoglobin
d. Extracellular methaemoglobin
e. Haemosiderin
Intracellular methaemoglobin
Which of the following statements best describes the features of amyloid angiopathy? [B2 Q40]
a. Solitary, peripheral haemorrhagic focus in a hypertensive elderly patient
b. Solitary, central haemorrhagic focus in a normotensive young patient
c. Multiple, peripheral haemorrhagic foci in a normotensive elderly patient
d. Multiple, central haemorrhagic foci in a normotensive elderly patient
e. Multiple, central haemorrhagic foci in a hypertensive young patient
Multiple, peripheral haemorrhagic foci in a normotensive elderly patient
Multiple peripheral haemorrhages (corticomedullary junction) in normotensive elderly patients are suggestive of amyloid angiopathy. Hypertensive haemorrhage tends to occur centrally around the basal ganglia (putamen, external capsule), thalamus and pons.
Which of the following best describes the typical appearance of intracranial haemorrhage in the late subacute phase (8-30 days)? [B3 Q24]
A. Increased T1, increased T2
B. Increased T1, decreased T2
C. Isointense T1, decreased T2
D. Isointense T1, increased T2
E. Decreased T1, decreased T2
A
B represents early subacute haemorrhage, C represents acute (12-72 hours), D represents hyperacute (less than 12 hours) and E represents chronic haemorrhage (> 1 month).
A 62-year-old man with a history of falls and confusion undergoes MR of the brain. This demonstrates a subdural haematoma of high signal intensity on T1W images and of high signal intensity with a hypointense rim on T2W images. What is the most likely age of the haematoma? [B4 Q8]
a. <6 hours
b. 8–72 hours
c. 3 days to 1 week
d. 1 week to several months
e. several months to several years
1 week to several months
In the first 3–6 hours (hyperacute stage) following haemorrhage, the intact red cells contain mostly oxyhaemoglobin, which appears hyperintense on T2W images. Desaturation occurs peripherally, forming deoxyhaemoglobin, which is seen as hypointensity on T2W images. In the acute stage (8–72 hours), there is rapid deoxygenation of the oxyhaemoglobin to deoxyhaemoglobin, which, together with the high protein content of the clot and susceptibility effects, results in iso- to hypointensity on T1W images and hypointensity on T2W images. In the early subacute stage (3–7 days), oxidation of deoxyhaemoglobin to methaemoglobin occurs inside the red cell, resulting in characteristic hyperintensity on T1W images due to paramagnetic effects, and marked hypointensity on T2W images. In the late subacute stage (1 week to months), extracellular methaemoglobin results in persistent hyperintensity on T1W images, but increasing signal intensity on T2W images, with peripheral susceptibility effects causing a low intensity rim. In the chronic stage (months to years), iron atoms are deposited as haemosiderin and ferritin, which cause susceptibility effects resulting in low signal intensity on both T1W and T2W images.
Which of the following imaging features of an intraparenchymal haemorrhage is most likely to suggest that the haemorrhage originates from an underlying tumour? [B4 Q75]
a. homogeneous CT appearance
b. complete haemosiderin ring
c. complete resolution on follow-up imaging
d. solitary haemorrhage
e. post-contrast enhancement of non-haemorrhagic areas
Post-contrast enhancement of non-haemorrhagic areas
Haemorrhage occurs in 5–10% of tumours. Features suggesting tumour rather than benign haemorrhage include complex CT pattern, incomplete haemosiderin ring, persisting oedema or mass on follow-up studies, and multiple lesions.
A hypertensive, 75-year-old female admitted with an acute stroke is found on unenhanced CT of the brain, to have an acute basal ganglia haemorrhage. If an MRI were performed 2 weeks later, what signal characteristics would the region of haemorrhage return? [B4 Q79]
a. isointense on T1W images and hyperintense on T2W images
b. hyperintense on T1W images and hyperintense on T2W images
c. isointense on T1W images and hypointense on T2W images
d. hyperintense on T1W images and hypointense on T2W images
e. hypointense on T1W images and a hypointense rim on T2W images
Hyperintense on T1W images and hyperintense on T2W images
The MRI signal of blood depends first on whether it is moving or static, since on most sequences movement produces a signal void. When it is static, the signal returned by blood reflects the magnetic properties of the blood products and their location. Hyperacute haemorrhage is intracellular oxyhaemoglobin that is diamagnetic, returning an isointense T1 and bright T2 signal. At 1–2 days, deoxygenation has occurred, making the iron paramagnetic. It remains intracellular and returns an isointense T1 signal and is dark on T2W images. At 2–7 days, haemorrhage contains paramagnetic intracellular methaemoglobin. This is bright on T1W images and dark on T2W images. The methaemoglobin becomes extracellular from 1 week to 4 weeks, and the MRI signal is bright on both T1W and T2W sequences. Chronic haemorrhage contains haemosiderin/ferritin, which is ferromagnetic, is dark on T1W images and has a dark rim on T2W sequences.
A 34-year-old man undergoes MRI of brain after admission for head trauma. Which of the following sequences is most sensitive for subarachnoid haemorrhage?
A. T1WI.
B. T1WI with fat saturation.
C. T2WI.
D. FLAIR.
E. Proton density.
FLAIR.
Although CT is generally used for investigating acute subarachnoid haemorrhage (SAH), FLAIR sequence on MRI has been suggested as being as sensitive as or more sensitive than CT. It is particularly useful in regions where CT may be limited due to beam hardening artefacts or if there is a very small amount of blood. Acute SAH appears as high intensity on FLAIR within the cisterns and sulci. Subacute SAH may be better appreciated on MRI because of its high signal intensity when the blood is isointense to CSF on CT. SAH differs from intra-parenchymal haemorrhage in that the mix of blood with high-oxygen tension CSF delays generation of paramagnetic deoxyhaemoglobin, and oxyhaemoglobin remains present longer than in intra-parenchymal haemorrhage. This contributes to continued T2 prolongation. Beware that there are other pathological (meningitis, leptomeningeal metastases, acute stroke, fat-containing tumour/dermoid rupture) and benign (artefact, supplemental oxygenation) causes of FLAIR hyperintensity in the subarachnoid space. Chronic haemorrhage from SAH is best detected on GE sequences, resulting in marked subarachnoid low signal (the blooming of superficial siderosis).
A 72-year-old man from a nursing home presents to the accident and emergency department. Nurses have noticed increasing confusion following a fall six days earlier. His inflammatory markers are normal. A non-contrast CT scan of the head demonstrates a crescent-shaped collection in the left fronto-parietal region. The collection is iso-dense to CSF and there is no midline shift, nor hydrocephalus. On T1-weighted MR imaging the lesion is isointense to CSF. The most likely diagnosis is: [B2 Q19]
a. Subdural hygroma
b. Brain atrophy
c. Subdural empyema
d. Chronic subdural haematoma
e. Enlarged subarachnoid space
Subdural hygroma
This is a traumatic subdural effusion which shows up as a localised CSF-fluid collection within the subdural space. They present in the elderly or in young children usually 6–30 days following trauma. The majority are asymptomatic, but patients may present with increasing confusion or headaches. They are devoid of blood products on imaging, unlike chronic subdural haematomas. Subdural haematomas are also more likely to cause effacement of the ventricular system and loss of the normal sulci-gyral pattern. Normal inflammatory markers and lack of pyrexia lessen the probability of an empyema.